Preview (15 of 104 pages)

NCLEX RN ACTUAL EXAM TEST BANK OF REAL QUESTIONS &
ANSWERS
NCLEX 2023/2024 graded A
Part 1
QUESTION 1
Which classification of drugs is contraindicated for the client with hypertrophic
cardiomyopathy?
A. Positive inotropes
B. Vasodilators
C. Diuretics
D. Anti dysrhythmics
Answer: A
Explanation:
(A) Positive inotropic agents should not be administered owing to their action of increasing
myocardial contractility. Increased ventricular contractility would increase outflow tract
obstruction in the client with hypertrophic cardiomyopathy.
(B) Vasodilators are not typically prescribed but are not contraindicated.
(C) Diuretics are used with caution to avoid causing hypovolemia.
(D) Anti dysrhythmics are typically needed to treat both atrial and ventricular dysrhythmias.
QUESTION 2
Signs and symptoms of an allergy attack include which of the following?
A. Wheezing on inspiration
B. Increased respiratory rate
C. Circumoral cyanosis
D. Prolonged expiration
Answer: D
Explanation:
(A) Wheezing occurs during expiration when air movement is impaired because of
constricted edematous bronchial lumina.
(B) Respirations are difficult, but the rate is frequently normal.
(C) The circumoral area is usually pale. Cyanosis is not an early sign of hypoxia.

(D) Expiration is prolonged because the alveoli are greatly distended and air trapping occurs.
QUESTION 3
A client confides to the nurse that he tasted poison in his evening meal. This would be an
example of what type of hallucination?
A. Auditory
B. Gustatory
C. Olfactory
D. Visceral
Answer: B
Explanation:
(A) Auditory hallucinations involve sensory perceptions of hearing.
(B) Gustatory hallucinations involve sensory perceptions of taste.
(C) Olfactory hallucinations involve sensory perceptions of smell.
(D) Visceral hallucinations involve sensory perceptions of sensation.
QUESTION 4
Which of the following findings would be abnormal in a post partial woman?
A. Chills shortly after delivery
B. Pulse rate of 60 bpm in morning on first postdelivery day
C. Urinary output of 3000 mL on the second day after delivery
D. An oral temperature of 101°F (38.3°C) on the third day after delivery
Answer: D
Explanation:
(A) Frequently the mother experiences a shaking chill immediately after delivery, which is
related to a nervous response or to vasomotor changes. If not followed by a fever, it is
clinically innocuous.
(B) The pulse rate during the immediate post partal period may be low but presents no cause
for alarm. The body attempts to adapt to the decreased pressures intra-abdominally as well as
from the reduction of blood flow to the vascular bed.
(C) Urinary output increases during the early post partal period (12–24 hours) owing to
diuresis. The kidneys must eliminate an estimated 2000–3000 mL of extracellular fluid
associated with a normal pregnancy.

(D) A temperature of 100.4°F (38°C) may occur after delivery as a result of exertion and
dehydration of labor. However, any temperature greater than 100.4°F needs further
investigation to identify any infectious process.
QUESTION 5
A six-month-old infant has been admitted to the emergency room with febrile seizures. In the
teaching of the parents, the nurse states that:
A. Sustained temperature elevation over 103°F is generally related to febrile seizures
B. Febrile seizures do not usually recur
C. There is little risk of neurological deficit and mental retardation as sequelae to febrile
seizures
D. Febrile seizures are associated with diseases of the central nervous system
Answer: C
Explanation:
(A) The temperature elevation related to febrile seizures generally exceeds 101°F, and
seizures occur during the temperature rise rather than after a prolonged elevation.
(B) Febrile seizures may recur and are more likely to do so when the first seizure occurs in
the 1st year of life.
(C) There is little risk of neurological deficit, mental retardation, or altered behavior
secondary to febrile seizures.
(D) Febrile seizures are associated with disease of the central nervous system.
QUESTION 6
A client diagnosed with bipolar disorder continues to be hyperactive and to lose weight.
Which of the following nutritional interventions would be most therapeutic for him at this
time?
A. Small, frequent feedings of foods that can be carried
B. Tube feedings with nutritional supplements
C. Allowing him to eat when and what he wants
D. Giving him a quiet place where he can sit down to eat meals
Answer: A
Explanation:
(A) The manic client is unable to sit still long enough to eat an adequate meal. Small, frequent
feedings with finger foods allow him to eat during periods of activity.

(B) This type of therapy should be implemented when other methods have been exhausted.
(C) The manic client should not be in control of his treatment plan. This type of client may
forget to eat.
(D) The manic client is unable to sit down to eat full meals.
QUESTION 7
A client with bipolar disorder taking lithium tells the nurse that he has ringing in his ears,
blurred vision, and diarrhea. The nurse notices a slight tremor in his left hand and a slurring
pattern to his speech. Which of the following actions by the nurse is appropriate?
A. Administer a stat dose of lithium as necessary.
B. Recognize this as an expected response to lithium.
C. Request an order for a stat blood lithium level.
D. Give an oral dose of lithium antidote.
Answer: C
Explanation:
(A) These symptoms are indicative of lithium toxicity. A stat dose of lithium could be fatal.
(B) These are toxic effects of lithium therapy.
(C) The client is exhibiting symptoms of lithium toxicity, which may be validated by lab
studies.
(D) There is no known lithium antidote.
QUESTION 8
A diagnosis of hepatitis C is confirmed by a male client’s physician. The nurse should be
knowledgeable of the differences between hepatitis A, B, and C. Which of the following are
characteristics of hepatitis C?
A. The potential for chronic liver disease is minimal.
B. The onset of symptoms is abrupt.
C. The incubation period is 2–26 weeks.
D. There is an effective vaccine for hepatitis B, but not for hepatitis C.
Answer: C
Explanation:
(A) Hepatitis C and B may result in chronic liver disease. Hepatitis A has a low potential for
chronic liver disease.
(B) Hepatitis C and B have insidious onsets. Hepatitis A has an abrupt onset.

(C) Incubation periods are as follows: hepatitis C is 2–26 weeks, hepatitis B is 6–20 weeks,
and hepatitis A is 2–6 weeks.
(D) Only hepatitis B has an effective vaccine.
QUESTION 9
Hypoxia is the primary problem related to near-drowning victims. The first organ that
sustains irreversible damage after submersion in water is the:
A. Kidney (urinary system)
B. Brain (nervous system)
C. Heart (circulatory system)
D. Lungs (respiratory system)
Answer: B
Explanation:
(A) The kidney can survive after 30 minutes of water submersion.
(B) The cerebral neurons sustain irreversible damage after 4–6 minutes of water submersion.
(C) The heart can survive up to 30 minutes of water submersion.
(D) The lungs can survive up to 30 minutes of water submersion.
QUESTION 10
Which of the following activities would be most appropriate during occupational therapy for
a client with bipolar disorder?
A. Playing cards with other clients
B. Working crossword puzzles
C. Playing tennis with a staff member
D. Sewing beads on a leather belt
Answer: C
Explanation:
(A) This activity is too competitive, and the manic client might become abusive toward the
other clients.
(B) During mania, the client’s attention span is too short to accomplish this task.
(C) This activity uses gross motor skills, eases tension, and expands excess energy. A staff
member is better equipped to interact therapeutically with clients.
(D) This activity requires the use of fine motor skills and is very tedious.

QUESTION 11
A 30-year-old male client is admitted to the psychiatric unit with a diagnosis of bipolar
disorder. For the last 2 months, his family describes him as being “on the move”, sleeping 3–
4 hours nightly, spending lots of money, and losing approximately 10 lb. During the initial
assessment with the client, the nurse would expect him to exhibit which of the following?
A. Short, polite responses to interview questions
B. Introspection related to his present situation
C. Exaggerated self-importance
D. Feelings of helplessness and hopelessness
Answer: C
Explanation:
(A) During the manic phase of bipolar disorder, clients have short attention spans and may be
abusive toward authority figures.
(B) Introspection requires focusing and concentration; clients with mania experience flight of
ideas, which prevents concentration.
(C) Grandiosity and an inflated sense of self-worth are characteristic of this disorder.
(D) Feelings of helplessness and hopelessness are symptoms of the depressive stage of
bipolar disorder.
QUESTION 12
Diabetes during pregnancy requires tight metabolic control of glucose levels to prevent
perinatal mortality. When evaluating the pregnant client, the nurse knows the recommended
serum glucose range during pregnancy is:
A. 70 mg/dL and 120 mg/dL
B. 100 mg/dL and 200 mg/dL
C. 40 mg/dL and 130 mg/dL
D. 90 mg/dL and 200 mg/dL
Answer: A
Explanation:
(A) The recommended range is 70–120 mg/dL to reduce the risk of perinatal mortality.
(B, C, D) These levels are not recommended. The higher the blood glucose, the worse the
prognosis for the fetus. Hypoglycemia can also have detrimental effects on the fetus.

QUESTION 13
When evaluating a client with symptoms of shock, it is important for the nurse to differentiate
between neurogenic and hypovolemic shock. The symptoms of neurogenic shock differ from
hypovolemic shock in that:
A. In neurogenic shock, the skin is warm and dry
B. In hypovolemic shock, there is a bradycardia
C. In hypovolemic shock, capillary refill is less than 2 seconds
D. In neurogenic shock, there is delayed capillary refill
Answer: A
Explanation:
(A) Neurogenic shock is caused by injury to the cervical region, which leads to loss of
sympathetic control. This loss leads to vasodilation of the vascular beds, bradycardia resulting
from the lack of sympathetic balance to parasympathetic stimuli from the vagus nerve, and
the loss of the ability to sweat below the level of injury. In neurogenic shock, the client is
hypotensive but brady cardiac with warm, dry skin.
(B) In hypovolemic shock, the client is hypotensive and tachycardiac with cool skin.
(C) In hypovolemic shock, the capillary refill would be 5 seconds.
(D) In neurogenic shock, there is no capillary delay, the vascular beds are dilated, and
peripheral flow is good.
QUESTION 14
A 55-year-old man is admitted to the hospital with complaints of fatigue, jaundice, anorexia,
and clay-colored stools. His admitting diagnosis is “rule out hepatitis”. Laboratory studies
reveal elevated liver enzymes and bilirubin. In obtaining his health history, the nurse should
assess his potential for exposure to hepatitis. Which of the following represents a high-risk
group for contracting this disease?
A. Heterosexual males
B. Oncology nurses
C. American Indians
D. Jehovah’s Witnesses
Answer: B
Explanation:
(A) Homosexual males, not heterosexual males, are at high risk for contracting hepatitis.

(B) Oncology nurses are employed in high-risk areas and perform invasive procedures that
expose them to potential sources of infection.
(C) The literature does not support the idea that any ethnic groups are at higher risk.
(D) There is no evidence that any religious groups are at higher risk.
QUESTION 15
A schizophrenic client has made sexual overtures toward her physician on numerous
occasions. During lunch, the client tells the nurse, “My doctor is in love with me and wants to
marry me.” This client is using which of the following defense mechanisms?
A. Displacement
B. Projection
C. Reaction formation
D. Suppression
Answer: B
Explanation:
(A) Displacement involves transferring feelings to a more acceptable object.
(B) Projection involves attributing one’s thoughts or feelings to another person.
(C) Reaction formation involves transforming an unacceptable impulse into the opposite
behavior.
(D) Suppression involves the intentional exclusion of unpleasant thoughts or experiences.
QUESTION 16
When teaching a sex education class, the nurse identifies the most common STDs in the
United States as:
A. Chlamydia
B. Herpes genitalis
C. Syphilis
D. Gonorrhea
Answer: A
Explanation:
(A) Chlamydia trachomatis infection is the most common STD in the United States. The
Centers for Disease Control and Prevention recommend screening of all high-risk women,
such as adolescents and women with multiple sex partners.

(B) Herpes simplex genitalia is estimated to be found in 5–20 million people in the United
States and is rising in occurrence yearly.
(C) Syphilis is a chronic infection caused by Treponema pallidum. Over the last several years
the number of people infected has begun to increase.
(D) Gonorrhea is a bacterial infection caused by the organism Neisseria gonorrhoeae.
Although gonorrhea is common, chlamydia is still the most common STD.
QUESTION 17
The nurse assists a client with advanced emphysema to the bathroom. The client becomes
extremely short of breath while returning to bed. The nurse should:
A. Increase his nasal O2 to 6 L/min
B. Place him in a lateral Sims position
C. Encourage pursed-lip breathing
D. Have him breathe into a paper bag
Answer: C
Explanation:
(A) Giving too high a concentration of O2 to a client with emphysema may remove his
stimulus to breathe.
(B) The client should sit forward with his hands on his knees or an overbed table and with
shoulders elevated.
(C) Pursed-lip breathing helps the client to blow off CO2 and to keep air passages open.
(D) Covering the face of a client extremely short of breath may cause anxiety and further
increase dyspnea.
QUESTION 18
In a client with chest trauma, the nurse needs to evaluate mediastinal position. This can best
be done by:
A. Auscultating bilateral breath sounds
B. Palpating for presence of crepitus
C. Palpating for trachial deviation
D. Auscultating heart sounds
Answer: C
Explanation:
(A) No change in the breath sounds occurs as a direct result of the mediastinal shift.

(B) Crepitus can occur owing to the primary disorder, not to the mediastinal shift.
(C) Mediastinal shift occurs primarily with tension pneumothorax, but it can occur with very
large hemothorax or pneumothorax. Mediastinal shift causes trachial deviation and deviation
of the heart’s point of maximum impulse.
(D) No change in the heart sounds occurs as a result of the mediastinal shift.
QUESTION 19
Clinical manifestations seen in left-sided rather than in right-sided heart failure are:
A. Elevated central venous pressure and peripheral edema
B. Dyspnea and jaundice
C. Hypotension and hepatomegaly
D. Decreased peripheral perfusion and rales
Answer: D
Explanation:
(A, B, C) Clinical manifestations of right-sided heart failure are weakness, peripheral edema,
jugular venous distention, hepatomegaly, jaundice, and elevated central venous pressure.
(D) Clinical manifestations of left- sided heart failure are left ventricular dysfunction,
decreased cardiac output, hypotension, and the backward failure as a result of increased left
atrium and pulmonary artery pressures, pulmonary edema, and rales.
QUESTION 20
In assessing cardiovascular clients with progression of aortic stenosis, the nurse should be
aware that there is typically:
A. Decreased pulmonary blood flow and cyanosis
B. Increased pressure in the pulmonary veins and pulmonary edema
C. Systemic venous engorgement
D. Increased left ventricular systolic pressures and hypertrophy
Answer: D
Explanation:
(A) These signs are seen in pulmonic stenosis or in response to pulmonary congestion and
edema and mitral stenosis.
(B) These signs are seen primarily in mitral stenosis or as a late sign in aortic stenosis after
left ventricular failure.
(C) These signs are seen primarily in right-sided heart valve dysfunction.

(D) Left ventricular hypertrophy occurs to increase muscle mass and overcome the stenosis;
left ventricular pressures increase as left ventricular volume increases owing to insufficient
emptying.
QUESTION 21
An 8-year-old child comes to the physician’s office complaining of swelling and pain in the
knees. His mother says, “The swelling occurred for no reason, and it keeps getting worse.”
The initial diagnosis is Lyme disease. When talking to the mother and child, questions related
to which of the following would be important to include in the initial history?
A. A decreased urinary output and flank pain
B. A fever of over 103°F occurring over the last 2–3 weeks
C. Rashes covering the palms of the hands and the soles of the feet
D. Headaches, malaise, or sore throat
Answer: D
Explanation:
(A) Urinary tract symptoms are not commonly associated with Lyme disease.
(B) A fever of 103°F is not characteristic of Lyme disease.
(C) The rash that is associated with Lyme disease does not appear on the palms of the hands
and the soles of the feet.
(D) Classic symptoms of Lyme disease include headache, malaise, fatigue, anorexia, stiff
neck, generalized lymphadenopathy, splenomegaly, conjunctivitis, sore throat, abdominal
pain, and cough.
QUESTION 22
When administering phenytoin (Dilantin) to a child, the nurse should be aware that a toxic
effect of phenytoin therapy is:
A. Stephens-Johnson syndrome
B. Folate deficiency
C. Leukopenic aplastic anemia
D. Granulocytosis and nephrosis
Answer: A
Explanation:
(A) Stephens-Johnson syndrome is a toxic effect of phenytoin.
(B) Folate deficiency is a side effect of phenytoin, but not a toxic effect.

(C) Leukopenic aplastic anemia is a toxic effect of carbamazepine (Tegretol).
(D) Granulocytosis and nephrosis are toxic effects of trimethadione (Tridione).
QUESTION 23
The nurse should know that according to current thinking, the most important prognostic
factor for a client with breast cancer is:
A. Tumor size
B. Axillary node status
C. Client’s previous history of disease
D. Client’s level of estrogen-progesterone receptor assays
Answer: B
Explanation:
(A) Although tumor size is a factor in classification of cancer growth, it is not an indicator of
lymph node spread.
(B) Axillary node status is the most important indicator for predicting how far the cancer has
spread. If the lymph nodes are positive for cancer cells, the prognosis is poorer.
(C) The client’s previous history of cancer puts her at an increased risk for breast cancer
recurrence, especially if the cancer occurred in the other breast. It does not predict prognosis,
however.
(D) The estrogen-progesterone assay test is used to identify present tumors being fed from an
estrogen site within the body. Some breast cancers grow rapidly as long as there is an
estrogen supply such as from the ovaries. The estrogen-progesterone assay test does not
indicate the prognosis.
QUESTION 24
Three weeks following discharge, a male client is readmitted to the psychiatric unit for
depression. His wife stated that he had threatened to kill himself with a handgun. As the nurse
admits him to the unit, he says, “I wish I were dead because I am worthless to everyone; I
guess I am just no good.” Which response by the nurse is most appropriate at this time?
A. “I don’t think you are worthless. I’m glad to see you, and we will help you.”
B. “Don’t you think this is a sign of your illness?”
C. “I know with your wife and new baby that you do have a lot to live for.”
D. “You’ve been feeling sad and alone for some time now?”
Answer: D

Explanation:
(A) This response does not acknowledge the client’s feelings.
(B) This is a closed question and does not encourage communication.
(C) This response negates the client’s feelings and does not require a response from the client.
(D) This acknowledges the client’s implied thoughts and feelings and encourages a response.
QUESTION 25
Which of the following should be included in discharge teaching for a client with hepatitis C?
A. He should take aspirin as needed for muscle and joint pain.
B. He may become a blood donor when his liver enzymes return to normal.
C. He should avoid alcoholic beverages during his recovery period.
D. He should use disposable dishes for eating and drinking.
Answer: C
Explanation:
(A) Aspirin is hepatotoxic, may increase bleeding, and should be avoided.
(B) Blood should not be donated by a client who has had hepatitis C because of the
possibility of transmission of disease.
(C) Alcohol is detoxified in the liver.
(D) Hepatitis C is not spread through the oral route.
QUESTION 26
The initial treatment for a client with a liquid chemical burn injury is to:
A. Irrigate the area with neutralizing solutions
B. Flush the exposed area with large amounts of water
C. Inject calcium chloride into the burned area
D. Apply lanolin ointment to the area
Answer: B
Explanation:
(A) In the past, neutralizing solutions were recommended, but presently there is concern that
these solutions extend the depth of burn.
(B) The use of large amounts of water to flush the area is recommended for chemical burns.
(C) Calcium chloride is not recommended therapy and would likely worsen the problem.
(D) Lanolin is of no benefit in the initial treatment of a chemical injury and may actually
extend a thermal injury.

QUESTION 27
Dietary planning is an essential part of the diabetic client’s regimen. The American Diabetes
Association recommends which of the following caloric guidelines for daily meal planning?
A. 50% complex carbohydrate, 20%–25% protein, 20%–25% fat
B. 45% complex carbohydrate, 25%–30% protein, 30%–35% fat
C. 70% complex carbohydrate, 20%–30% protein, 10%–20% fat
D. 60% complex carbohydrate, 12%–15% protein, 20%–25% fat
Answer: D
Explanation:
(A) The percentage of carbohydrates is too low to maintain blood sugar levels. The percent
range of protein is too high and may cause extra workload on the kidney as it is metabolized.
(B) The percentage of carbohydrates is too low to maintain blood sugar levels. The percent
range of protein is too high and may cause extra workload on the kidney.
(C) The percentage of carbohydrates is too high; the percent range of protein is too high, and
of fat, too low.
(D) This combination provides enough carbohydrates to maintain blood glucose levels,
enough protein to maintain body repair, and enough fat to ensure palatability.
QUESTION 28
The primary reason for sending a burn client home with a pressure garment, such as a Jobst
garment, is that the garment:
A. Decreases hypertrophic scar formation
B. Assists with ambulation
C. Covers burn scars and decreases the psychological impact during recovery
D. Increases venous return and cardiac output by normalizing fluid status
Answer: A
Explanation:
(A) Tubular support, such as that received with a Jobst garment, applies tension of 10–20 mm
Hg. This amount of uniform pressure is necessary to prevent or reduce hypertrophic scarring.
Clients typically wear a pressure garment for 6–12 months during the recovery phase of their
care.
(B) Pressure garments have no ambulatory assistive properties.

(C) Pressure garments can worsen the psychological impact of burn injury, especially if worn
on the face.
(D) Pressure garments do not normalize fluid status.
QUESTION 29
The nurse would expect to include which of the following when planning the management of
the client with Lyme disease?
A. Complete bed rest for 6–8 weeks
B. Tetracycline treatment
C. IV amphotericin B
D. High-protein diet with limited fluids
Answer: B
Explanation:
(A) The client is not placed on complete bed rest for 6 weeks.
(B) Tetracycline is the treatment of choice for children with Lyme disease who are over the
age of 9.
(C) IV amphotericin B is the treatment for histoplasmosis.
(D) The client is not restricted to a high-protein diet with limited fluids.
QUESTION 30
The physician recommends immediate hospital admission for a client with PIH. She says to
the nurse, “It’s not so easy for me to just go right to the hospital like that.” After
acknowledging her feelings, which of these approaches by the nurse would probably be best?
A. Stress to the client that her husband would want her to do what is best for her health.
B. Explore with the client her perceptions of why she is unable to go to the hospital.
C. Repeat the physician’s reasons for advising immediate hospitalization.
D. Explain to the client that she is ultimately responsible for her own welfare and that of her
baby.
Answer: B
Explanation:
(A) This answer does not hold the client accountable for her own health.
(B) The nurse should explore potential reasons for the client’s anxiety: are there small
children at home, is the husband out of town? The nurse should aid the client in seeking
support or interventions to decrease the anxiety of hospitalization.

(C) Repeating the physician’s reason for recommending hospitalization may not aid the client
in dealing with her reasons for anxiety.
(D) The concern for self and welfare of baby may be secondary to a woman who is in a crisis
situation. The nurse should explore the client’s potential reasons for anxiety. For example, is
there another child in the home who is ill, or is there a husband who is overseas and not able
to return on short notice?
QUESTION 31
The child with iron poisoning is given IV deforoxamine mesylate (Desferal). Following
administration, the child suffers hypotension, facial flushing, and urticaria. The initial nursing
intervention would be to:
A. Discontinue the IV
B. Stop the medication, and begin a normal saline infusion
C. Take all vital signs, and report to the physician
D. Assess urinary output, and if it is 30 mL an hour, maintain current treatment
Answer: B
Explanation:
(A) The IV line should not be discontinued because other IV medications will be needed.
(B) Stop the medication and begin a normal saline infusion. The child is exhibiting signs of
an allergic reaction and could go into shock if the medication is not stopped. The line should
be kept opened for other medication.
(C) Taking vital signs and reporting to the physician is not an adequate intervention because
the IV medication continues to flow.
(D) Assessing urinary output and, if it is 30 mL an hour, maintaining current treatment is an
inappropriate intervention owing to the child’s obvious allergic reaction.
QUESTION 32
Provide the 1-minute Apgar score for an infant born with the following findings: Heart rate:
Above 100 Respiratory effort: Slow, irregular Muscle tone: Some flexion of extremities
Reflex irritability: Vigorous cry Color: Body pink, blue extremities
A. 7
B. 10
C. 8
D. 9

Answer: A
Explanation:
(A) Seven out of a possible perfect score of 10 is correct. Two points are given for heart rate
above 100; 1 point is given for slow, irregular respiratory effort; 1 point is given for some
flex- ion of extremities in assessing muscle tone; 2 points are given for vigorous cry in
assessing reflex irritability; 1 point is assessed for color when the body is pink with blue
extremities (acrocyanosis).
(B) For a perfect Apgar score of 10, the infant would have a heart rate over 100 but would
also have a good cry, active motion, and be completely pink.
(C) For an Apgar score of 8 the respiratory rate, muscle tone, or color would need to fall into
the 2-point rather than the 1-point category.
(D) For this infant to receive an Apgar score of 9, four of the areas evaluated would need
ratings of 2 points and one area, a rating of 1 point.
QUESTION 33
A client has been diagnosed as being preeclamptic. The physician orders magnesium sulfate.
Magnesium sulfate (MgSO4) is used in the management of preeclampsia for:
A. Prevention of seizures
B. Prevention of uterine contractions
C. Sedation
D. Fetal lung protection
Answer: A
Explanation:
(A) MgSO4 is classified as an anticonvulsant drug. In preeclampsia management, MgSO4 is
used for prevention of seizures.
(B) MgSO4 has been used to inhibit hyperactive labor, but results are questionable.
(C) Negative side effects such as respiratory depression should not be confused with
generalized sedation.
(D) MgSO4 does not affect lung maturity. The infant should be assessed for neuromuscular
and respiratory depression.
QUESTION 34
In the client with a diagnosis of coronary artery disease, the nurse would anticipate the
complication of bradycardia with occlusion of which coronary artery?

A. Right coronary artery
B. Left main coronary artery
C. Circumflex coronary artery
D. Left anterior descending coronary artery
Answer: A
Explanation:
(A) Sinus bradycardia and atrioventricular (AV) heart block are usually a result of right
coronary artery occlusion. The right coronary artery perfuses the sinoatrial and AV nodes in
most individuals.
(B) Occlusion of the left main coronary artery causes bundle branch blocks and premature
ventricular contractions.
(C) Occlusion of the circumflex artery does not cause bradycardia.
(D) Sinus tachycardia occurs primarily with left anterior descending coronary artery
occlusion because this form of occlusion impairs left ventricular function.
QUESTION 35
When the nurse is evaluating lab data for a client 18–24 hours after a major thermal burn, the
expected physiological changes would include which of the following?
A. Elevated serum sodium
B. Elevated serum calcium
C. Elevated serum protein
D. Elevated hematocrit
Answer: D
Explanation:
(A) Sodium enters the edema fluid in the burned area, lowering the sodium content of the
vascular fluid. Hyponatremia may continue for days to several weeks because of sodium loss
to edema, sodium shifting into the cells, and later, diuresis.
(B) Hypocalcemia occurs because of calcium loss to edema fluid at the burned site (third
space fluid).
(C) Protein loss occurs at the burn site owing to increased capillary permeability. Serum
protein levels remain low until healing occurs.
(D) Hematocrit level is elevated owing to hemoconcentration from hypovolemia. Anemia is
present in the postburn stage owing to blood loss and hemolysis, but it cannot be assessed
until the client is adequately hydrated.

QUESTION 36
What is the most effective method to identify early breast cancer lumps?
A. Mammograms every 3 years
B. Yearly checkups performed by physician
C. Ultrasounds every 3 years
D. Monthly breast self-examination
Answer: D
Explanation:
(A) Mammograms are less effective than breast self-examination for the diagnosis of
abnormalities in younger women, who have denser breast tissue. They are more effective for
women older than 40.
(B) Up to 15% of early-stage breast cancers are detected by physical examination; however,
95% are detected by women doing breast self-examination.
(C) Ultrasound is used primarily to determine the location of cysts and to distinguish cysts
from solid masses.
(D) Monthly breast self-examination has been shown to be the most effective method for
early detection of breast cancer. Approximately 95% of lumps are detected by women
themselves.
QUESTION 37
A client with a C-3–4 fracture has just arrived in the emergency room. The primary nursing
intervention is:
A. Stabilization of the cervical spine
B. Airway assessment and stabilization
C. Confirmation of spinal cord injury
D. Normalization of intravascular volume
Answer: B
Explanation:
(A) If cervical spine injury is suspected, the airway should be maintained using the jaw thrust
method that also protects the cervical spine.
(B) Primary intervention is protection of the airway and adequate ventilation.
(C, D) All other interventions are secondary to adequate ventilation.

QUESTION 38
To ensure proper client education, the nurse should teach the client taking SL nitroglycerin to
expect which of the following responses with administration?
A. Stinging, burning when placed under the tongue
B. Temporary blurring of vision
C. Generalized urticaria with prolonged use
D. Urinary frequency
Answer: A
Explanation:
(A) Stinging or burning when nitroglycerin is placed under the tongue is to be expected. This
effect indicates that the medication is potent and effective for use. Failure to have this
response means that the client needs to get a new bottle of nitroglycerin.
(B, C, D) The other responses are not expected in this situation and are not even side effects.
QUESTION 39
Nursing care for the substance abuse client experiencing alcohol withdrawal delirium
includes:
A. Maintaining seizure precautions
B. Restricting fluid intake
C. Increasing sensory stimuli
D. Applying ankle and wrist restraints
Answer: A
Explanation:
(A) These clients are at high risk for seizures during the 1st week after cessation of alcohol
intake.
(B) Fluid intake should be increased to prevent dehydration.
(C) Environmental stimuli should be decreased to prevent precipitation of seizures.
(D) Application of restraints may cause the client to increase his or her physical activity and
may eventually lead to exhaustion.
QUESTION 40
The day following his admission, the nurse sits down by a male client on the sofa in the
dayroom. He was admitted for depression and thoughts of suicide. He looks at the nurse and

says, “My life is so bad no one can do anything to help me.” The most helpful initial response
by the nurse would be:
A. “It concerns me that you feel so badly when you have so many positive things in your
life.”
B. “It will take a few weeks for you to feel better, so you need to be patient.”
C. “You are telling me that you are feeling hopeless at this point?”
D. “Let’s play cards with some of the other clients to get your mind off your problems for
now.”
Answer: C
Explanation:
(A) This response does not acknowledge the client’s feelings and may increase his feelings of
guilt.
(B) This response denotes false reassurance.
(C) This response acknowledges the client’s feelings and invites a response.
(D) This response changes the subject and does not allow the client to talk about his feelings.
QUESTION 41
The usual treatment for diabetes insipidus is with IM or SC injection of vasopressin tannate in
oil. Nursing care related to the client receiving IM vasopressin tannate would include:
A. Weigh once a week and report to the physician any weight gain of 10 lb.
B. Limit fluid intake to 500 mL/day.
C. Store the medication in a refrigerator and allow to stand at room temperature for 30
minutes prior to administration.
D. Hold the vial under warm water for 10–15 minutes and shake vigorously before drawing
medication into the syringe.
Answer: D
Explanation:
(A) Weight should be obtained daily.
(B) Fluid is not restricted but is given according to urine output.
(C) The medication does not have to be stored in a refrigerator.
(D) Holding the vial under warm water for 10–15 minutes or rolling between your hands and
shaking vigorously before drawing medication into the syringe activates the medication in the
oil solution.

QUESTION 42
Proper positioning for the child who is in Bryant’s traction is:
A. Both hips flexed at a 90-degree angle with the knees extended and the buttocks elevated
off the bed
B. Both legs extended, and the hips are not flexed
C. The affected leg extended with slight hip flexion
D. Both hips and knees maintained at a 90-degree flexion angle, and the back flat on the bed
Answer: A
Explanation:
(A) The child’s weight supplies the countertraction for Bryant’s traction; the buttocks are
slightly elevated off the bed, and the hips are flexed at a 90-degree angle. Both legs are
suspended by skin traction.
(B) The child in Buck’s extension traction maintains the legs extended and parallel to the bed.
(C) The child in Russell traction maintains hip flexion of the affected leg at the prescribed
angle with the leg extended.
(D) The child in “90–90 traction” maintains both hips and knees at a 90-degree flexion angle
and the back is flat on the bed.
QUESTION 43
The predominant purpose of the first Apgar scoring of a newborn is to:
A. Determine gross abnormal motor function
B. Obtain a baseline for comparison with the infant’s future adaptation to the environment
C. Evaluate the infant’s vital functions
D. Determine the extent of congenital malformations
Answer: C
Explanation:
(A) Apgar scores are not related to the infant’s care, but to the infant’s physical condition.
(B) Apgar scores assess the current physical condition of the infant and are not related to
future environmental adaptation.
(C) The purpose of the Apgar system is to evaluate the physical condition of the newborn at
birth and to determine if there is an immediate need for resuscitation.
(D) Congenital malformations are not one of the areas assessed with Apgar scores.

QUESTION 44
Hematotympanum and otorrhea are associated with which of the following head injuries?
A. Basilar skull fracture
B. Subdural hematoma
C. Epidural hematoma
D. Frontal lobe fracture
Answer: A
Explanation:
(A) Basilar skull fractures are fractures of the base of the skull. Blood behind the eardrum or
blood or cerebrospinal fluid (CSF) leaking from the ear are indicative of a dural laceration.
Basilar skull fractures are the only type with these symptoms.
(B, C, D) These do not typically cause dural lacerations and CSF leakage.
QUESTION 45
Which of the following risk factors associated with breast cancer would a nurse consider most
significant in a client’s history?
A. Menarche after age 13
B. Nulliparity
C. Maternal family history of breast cancer
D. Early menopause
Answer: C
Explanation:
(A) Women who begin menarche late (after 13 years old) have a lower risk of developing
breast cancer than women who have begun earlier. Average age for menarche is 12.5 years.
(B) Women who have never been pregnant have an increased risk for breast cancer, but a
positive family history poses an even greater risk.
(C) A positive family history puts a woman at an increased risk of developing breast cancer. It
is recommended that mammography screening begin 5 years before the age at which an
immediate female relative was diagnosed with breast cancer.
(D) Early menopause decreases the risk of developing breast cancer.

QUESTION 46
A client returns for her 6-month prenatal checkup and has gained 10 lb in 2 months. The
results of her physical examination are normal. How does the nurse interpret the effectiveness
of the instruction about diet and weight control?
A. She is compliant with her diet as previously taught.
B. She needs further instruction and reinforcement.
C. She needs to increase her caloric intake.
D. She needs to be placed on a restrictive diet immediately.
Answer: B
Explanation:
(A) She is probably not compliant with her diet and exercise program. Recommended weight
gain during second and third trimesters is approximately 12 lb.
(B) Because of her excessive weight gain of 10 lb in 2 months, she needs re-evaluation of her
eating habits and reinforcement of proper dietary habits for pregnancy. A 2200-calorie diet is
recommended for most pregnant women with a weight gain of 27–30 lb over the 9-month
period. With rapid and excessive weight gain, PIH should also be suspected.
(C) She does not need to increase her caloric intake, but she does need to re-evaluate dietary
habits. Ten pounds in 2 months is excessive weight gain during pregnancy, and health
teaching is warranted.
(D) Restrictive dieting is not recommended during pregnancy.
QUESTION 47
The priority nursing goal when working with an autistic child is:
A. To establish trust with the child
B. To maintain communication with the family
C. To promote involvement in school activities
D. To maintain nutritional requirements
Answer: A
Explanation:
(A) The priority nursing goal when working with an autistic child is establishing a trusting
relationship.
(B) Maintaining a relationship with the family is important but having the trust of the child is
a priority.

(C) To promote involvement in school activities is inappropriate for a child who is autistic.
(D) Maintaining nutritional requirements is not the primary problem of the autistic child.
QUESTION 48
The nurse would need to monitor the serum glucose levels of a client receiving which of the
following medications, owing to its effects on glycogenolysis and insulin release?
A. Norepinephrine (Levophed)
B. Dobutamine (Dobutrex)
C. Propranolol (Inderal)
D. Epinephrine (Adrenalin)
Answer: D
Explanation:
(A) Norepinephrine’s side effects are primarily related to safe, effective care environment and
include decreased peripheral perfusion and bradycardia.
(B) Dobutamine’s side effects include increased heart rate and blood pressure, ventricular
ectopy, nausea, and headache.
(C) Propranolol’s side effects include elevated blood urea nitrogen, serum transaminase,
alkaline phosphatase, and lactic dehydrogenase.
(D) Epinephrine increases serum glucose levels by increasing glycogenolysis and inhibiting
insulin release. Prolonged use can elevate serum lactate levels, leading to metabolic acidosis,
increased urinary catecholamines, false elevation of blood urea nitrogen, and decreased
coagulation time.
QUESTION 49
When a client questions the nurse as to the purpose of exercise electrocardiography (ECG) in
the diagnosis of cardiovascular disorders, the nurse’s response should be based on the fact
that:
A. The test provides a baseline for further tests
B. The procedure simulates usual daily activity and myocardial performance
C. The client can be monitored while cardiac conditioning and heart toning are done
D. Ischemia can be diagnosed because exercise increases O2 consumption and demand
Answer: D
Explanation:
(A) The purpose of the study is not to provide a baseline for further tests.

(B) The test causes an increase in O2 demand beyond that required to perform usual daily
activities.
(C) Monitoring does occur, but the test is not for the purpose of cardiac toning and
conditioning.
(D) Exercise ECG, or stress testing, is designed to elevate the peripheral and myocardial
needs for O2 to evaluate the ability of the myocardium and coronary arteries to meet the
additional demands.
QUESTION 50
The following medications were noted on review of the client’s home medication profile.
Which of the medications would most likely potentiate or elevate serum digoxin levels?
A. KCl
B. Thyroid agents
C. Quinidine
D. Theophylline
Answer: C
Explanation:
(A) Hypokalemia can cause digoxin toxicity. Administration of KCl would prevent this.
(B) Thyroid agents decrease digoxin levels.
(C) Quinidine increases digoxin levels dramatically.
(D) Theophylline is not noted to have an effect on digoxin levels.
QUESTION 51
Which of the following statements relevant to a suicidal client is correct?
A. The more specific a client’s plan, the more likely he or she is to attempt suicide.
B. A client who is unsuccessful at a first suicide attempt is not likely to make future attempts.
C. A client who threatens suicide is just seeking attention and is not likely to attempt suicide.
D. Nurses who care for a client who has attempted suicide should not make any reference to
the word “suicide in order to protect the client’s ego.”
Answer: A
Explanation:
(A) This is a high-risk factor for potential suicide.
(B) A previous suicide attempt is a definite risk factor for subsequent attempts.
(C) Every threat of suicide should be taken seriously.

(D) The client should be asked directly about his or her intent to do bodily harm. The client is
never hurt by direct, respectful questions.
QUESTION 52
When assessing a child with diabetes insipidus, the nurse should be aware of the cardinal
signs of:
A. Anemia and vomiting
B. Polyuria and polydipsia
C. Irritability relieved by feeding formula
D. Hypothermia and azotemia
Answer: B
Explanation:
(A) Anemia and vomiting are not cardinal signs of diabetes insipidus.
(B) Polyuria and polydipsia are the cardinal signs of diabetes insipidus.
(C) Irritability relieved by feeding water, not formula, is a common sign, but not the cardinal
sign, of diabetes insipidus.
(D) Hypothermia and azotemia are signs, but not cardinal signs, of diabetes insipidus.
QUESTION 53
The physician orders fluoxetine (Prozac) for a depressed client. Which of the following
should the nurse remember about fluoxetine?
A. Because fluoxetine is a tricyclic antidepressant, it may precipitate a hypertensive crisis.
B. The therapeutic effect of the drug occurs 2–4 weeks after treatment is begun.
C. Foods such as aged cheese, yogurt, soy sauce, and bananas should not be eaten with this
drug.
D. Fluoxetine may be administered safely in combination with monoamine oxidase (MAO)
inhibitors.
Answer: B
Explanation:
(A) Fluoxetine is not a tricyclic antidepressant. It is an atypical antidepressant.
(B) This statement is true.
(C) These foods are high in tyramine and should be avoided when the client is taking MAO
inhibitors. Fluoxetine is not an MAO inhibitor.

(D) Fatal reactions have been reported in clients receiving fluoxetine in combination with
MAO inhibitors.
QUESTION 54
A child sustains a supracondylar fracture of the femur. When assessing for vascular injury, the
nurse should be alert for the signs of ischemia, which include:
A. Bleeding, bruising, and hemorrhage
B. Increase in serum levels of creatinine, alkaline phosphatase, and aspartate transaminase
C. Pain, pallor, pulselessness, paresthesia, and paralysis
D. Generalized swelling, pain, and diminished functional use with muscle rigidity and
crepitus
Answer: C
Explanation:
(A) Bleeding, bruising, and hemorrhage may occur due to injury but are not classic signs of
ischemia.
(B) An increase in serum levels of creatinine, alkaline phosphatase, and aspartate
transaminase is related to the disruption of muscle integrity.
(C) Classic signs of ischemia related to vascular injury secondary to long bone fractures
include the five “P’s”: pain, pallor, pulselessness, paresthesia, and paralysis.
(D) Generalized swelling, pain, and diminished functional use with muscle rigidity and
crepitus are common clinical manifestations of a fracture but not ischemia.
QUESTION 55
A client with a diagnosis of C-4 injury has been stabilized and is ready for discharge. Because
this client is at risk for autonomic dysreflexia, he and his family should be instructed to assess
for and report:
A. Dizziness and tachypnea
B. Circumoral pallor and light-headedness
C. Headache and facial flushing
D. Pallor and itching of the face and neck
Answer: C
Explanation:
(A) Tachypnea is not a symptom.
(B) Circumoral pallor is not a symptom.

(C) Autonomic dysreflexia is an uninhibited and exaggerated reflex of the autonomic nervous
system to stimulation, which results in vasoconstriction and elevated blood pressure.
(D) Pallor and itching are not symptoms.
QUESTION 56
A client with emphysema is placed on diuretics. In order to avoid potassium depletion as a
side effect of the drug therapy, which of the following foods should be included in his diet?
A. Celery
B. Potatoes
C. Tomatoes
D. Liver
Answer: B
Explanation:
(A) Celery is high in sodium.
(B) Potatoes are high in potassium.
(C) Tomatoes are high in sodium.
(D) Liver is high in iron.
QUESTION 57
Which of the following signs and symptoms indicates a tension pneumothorax as compared
to an open pneumothorax?
A. Ventilation-perfusion (V./Q.) mismatch
B. Hypoxemia and respiratory acidosis
C. Mediastinal tissue and organ shifting
D. Decreased tidal volume and tachypnea
Answer: C
Explanation:
(A, B, D) These occur in both tension pneumothorax and open pneumothorax.
(C) The tension pneumothorax acts like a one- way valve so that the pneumothorax increases
with each breath. Eventually, it occupies enough space to shift mediastinal tissue toward the
unaffected side away from the midline. Tracheal deviation, movement of point of maximum
impulse, and decreased cardiac output will occur. The other three options will occur in both
types of pneumothorax.

QUESTION 58
A 38-year-old pregnant woman visits her nurse practitioner for her regular prenatal checkup.
She is 30 week’s gestation. The nurse should be alert to which condition related to her age?
A. Iron-deficiency anemia
B. Sexually transmitted disease (STD)
C. Intrauterine growth retardation
D. Pregnancy-induced hypertension (PIH)
Answer: D
Explanation:
(A) Iron-deficiency anemia can occur throughout pregnancy and is not age related.
(B) STDs can occur prior to or during pregnancy and are not age related.
(C) Intrauterine growth retardation is an abnormal process where fetal development and
maturation are delayed. It is not age related.
(D) Physical risks for the pregnant client older than 35 include increased risk for PIH,
cesarean delivery, fetal and neonatal mortality, and trisomy.
QUESTION 59
A type I diabetic client is diagnosed with cellulitis in his right lower extremity. The nurse
would expect which of the following to be present in relation to his blood sugar level?
A. A normal blood sugar level
B. A decreased blood sugar level
C. An increased blood sugar level
D. Fluctuating levels with a predawn increase
Answer: C
Explanation:
(A) Blood sugar levels increase when the body responds to stress and illness.
(B) Blood sugar levels increase when the body responds to stress and illness.
(C) Hyperglycemia occurs because glucose is produced as the body responds to the stress and
illness of cellulitis.
(D) Blood sugar levels remain elevated as long as the body responds to stress and illness.
QUESTION 60
A laboratory technique specific for diagnosing Lyme disease is:
A. Polymerase chain reaction

B. Heterophil antibody test
C. Decreased serum calcium level
D. Increased serum potassium level
Answer: A
Explanation:
(A) Polymerase chain reaction is the laboratory technique specific for Lyme disease.
(B) Heterophil antibody test is used to diagnose mononucleosis.
(C) Lyme disease does not decrease the serum calcium level.
(D) Lyme disease does not increase the serum potassium level.
QUESTION 61
The cardiac client who exhibits the symptoms of disorientation, lethargy, and seizures may be
exhibiting a toxic reaction to:
A. Digoxin (Lanoxin)
B. Lidocaine (Xylocaine)
C. Quinidine gluconate or sulfate (Quinaglute, Quinidex)
D. Nitroglycerin IV (Tridil)
Answer: B
Explanation:
(A) Side effects of digoxin include headache, hypotension, AV block, blurred vision, and
yellow-green halos.
(B) Side effects of lidocaine include heart block, headache, dizziness, confusion, tremor,
lethargy, and convulsions.
(C) Side effects of quinidine include heart block, hepatotoxicity, thrombocytopenia, and
respiratory depression.
(D) Side effects of nitroglycerin include postural hypotension, headache, dizziness, and
flushing.
QUESTION 62
A 27-year-old man was diagnosed with type I diabetes 3 months ago. Two weeks ago he
complained of pain, redness, and tenderness in his right lower leg. He is admitted to the
hospital with a slight elevation of temperature and vague complaints of “not feeling well.” At
4:30 PM on the day of his admission, his blood glucose level is 50 mg; dinner will be served
at 5:00 PM. The best nursing action would be to:

A. Give him 3 tbsp of sugar dissolved in 4 oz of grape juice to drink
B. Ask him to dissolve three pieces of hard candy in his mouth
C. Have him drink 4 oz of orange juice
D. Monitor him closely until dinner arrives
Answer: C
Explanation:
(A) The combination of sugar and juice will increase the blood sugar beyond the normal
range.
(B) Concentrated sweets are not absorbed as fast as juice; consequently, they elevate the
blood sugar beyond the normal limit.
(C) Four ounces of orange juice will act immediately to raise the blood sugar to a normal
level and sustain it for 30 minutes until supper is served.
(D) There is an increased potential for the client’s blood sugar to decrease even further,
resulting in diabetic coma.
QUESTION 63
After 3 weeks of treatment, a severely depressed client suddenly begins to feel better and
starts interacting appropriately with other clients and staff. The nurse knows that this client
has an increased risk for:
A. Suicide
B. Exacerbation of depressive symptoms
C. Violence toward others
D. Psychotic behavior
Answer: A
Explanation:
(A) When the severely depressed client suddenly begins to feel better, it often indicates that
the client has made the decision to kill himself or herself and has developed a plan to do so.
(B) Improvement in behavior is not indicative of an exacerbation of depressive symptoms.
(C) The depressed client has a tendency for self-violence, not violence toward others.
(D) Depressive behavior is not always accompanied by psychotic behavior.
QUESTION 64
Discharge teaching was effective if the parents of a child with atopic dermatitis could state
the importance of:

A. Maintaining a high-humidified environment
B. Furry, soft stuffed animals for play
C. Showering 3–4 times a day
D. Wrapping hands in soft cotton gloves
Answer: D
Explanation:
(A) Maintaining a low-humidified environment.
(B) Avoiding furry, soft stuffed animals for play, which may increase symptoms of allergy.
(C) Avoiding showering, which irritates the dermatitis, and encouraging bathing 4 times a day
in colloid bath for temporary relief.
(D) Wrapping hands in soft cotton gloves to prevent skin damage during scratching.
QUESTION 65
Which of the following would differentiate acute from chronic respiratory acidosis in the
assessment of the trauma client?
A. Increased PaCO2
B. Decreased PaO2
C. Increased HCO3
D. Decreased base excess
Answer: C
Explanation:
(A) Increased CO2 will occur in both acute and chronic respiratory acidosis.
(B) Hypoxia does not determine acid-base status.
(C) Elevation of HCO3 is a compensatory mechanism in acidosis that occurs almost
immediately, but it takes hours to show any effect and days to reach maximum compensation.
Renal disease and diuretic therapy may impair the ability of the kidneys to compensate.
(D) Base excess is a no respiratory contributor to acid-base balance. It would increase to
compensate for acidosis.
QUESTION 66
When a client is receiving vasoactive therapy IV, such as dopamine (Intropin), and
extravasation occurs, the nurse should be prepared to administer which of the following
medications directly into the site?
A. Phentolamine (Regitine)

B. Epinephrine
C. Phenylephrine (Neo-Synephrine)
D. Sodium bicarbonate
Answer: A
Explanation:
(A) Phentolamine is given to counteract the-adrenergic effects that cause ischemia and
necrosis of local tissue.
(B) Epinephrine is an endogenous catecholamine that produces vasoconstriction and increases
heart rate and contractility.
(C) Phenylephrine causes constriction of arterioles of skin, mucous membranes, and viscera,
which in turn can cause ischemia and necrosis.
(D) Sodium bicarbonate is an alkalinizing agent that is incompatible with dopamine.
QUESTION 67
A male client receives 10 U of regular human insulin SC at 9:00 AM. The nurse would expect
peak action from this injection to occur at:
A. 9:30 AM
B. 10:30 AM
C. 12 noon
D. 4:00 PM
Answer: C
Explanation:
(A) This is too early for peak action to occur.
(B) This is too early for peak action to occur.
(C) Regular insulin peak action occurs 2–4 hours after administration.
(D) This is too late for peak action to occur.
QUESTION 68
As the nurse assesses a male adolescent with chlamydia, the nurse determines that a sign of
chlamydia is:
A. Enlarged penis
B. Secondary lymphadenitis
C. Epididymitis
D. Hepatomegaly

Answer: C
Explanation:
(A) An enlarged penis is not a sign of chlamydia.
(B) Secondary lymphadenitis is a complication of lymphogranuloma venereum.
(C) Untreated chlamydial infection can spread from the urethra, causing epididymitis, which
presents as a tender, scrotal swelling.
(D) Hepatomegaly is not a complication.
QUESTION 69
One of the most dramatic and serious complications associated with bacterial meningitis is
Waterhouse- Friderichsen syndrome, which is:
A. Peripheral circulatory collapse
B. Syndrome of inappropriate antiduretic hormone
C. Cerebral edema resulting in hydrocephalus
D. Auditory nerve damage resulting in permanent hearing loss
Answer: A
Explanation:
(A) Waterhouse-Friderichsen syndrome is peripheral circulatory collapse, which may result in
extensive and diffuse intravascular coagulation and thrombocytopenia resulting in death.
(B) Syndrome of inappropriate antidiuretic hormone is a complication of meningitis, but it is
not Waterhouse- Friderichsen syndrome.
(C) Cerebral edema resulting in hydrocephalus is a complication of meningitis, but it is not
Waterhouse-Friderichsen syndrome.
(D) Auditory nerve damage resulting in permanent hearing loss is a complication of
meningitis, but it is not Waterhouse- Friderichsen syndrome.
QUESTION 70
Priapism may be a sign of:
A. Altered neurological function
B. Imminent death
C. Urinary incontinence
D. Reproductive dysfunction
Answer: A
Explanation:

(A) Priapism in the trauma client is due to the neurological dysfunction seen in spinal cord
injury. Priapism is an abnormal erection of the penis; it may be accompanied by pain and
tenderness. This may disappear as spinal cord edema is relieved.
(B) Priapism is not associated with death.
(C) Urinary retention, rather than incontinence, may occur.
(D) Reproductive dysfunction may be a secondary problem.
QUESTION 71
When assessing fetal heart rate status during labor, the monitor displays late decelerations
with tachycardia and decreasing variability. What action should the nurse take?
A. Continue monitoring because this is a normal occurrence.
B. Turn client on right side.
C. Decrease IV fluids.
D. Report to physician or midwife.
Answer: D
Explanation:
(A) This is not a normal occurrence. Late decelerations need prompt intervention for
immediate infant recovery.
(B) To increase O2 perfusion to the unborn infant, the mother should be placed on her left
side.
(C) IV fluids should be increased, not decreased.
(D) Immediate action is warranted, such as reporting findings, turning mother on left side,
administering O2, discontinuing oxytocin (Pitocin), assessing maternal blood pressure and the
labor process, preparing for immediate cesarean delivery, and explaining plan of action to
client.
QUESTION 72
The most commonly known vectors of Lyme disease are:
A. Mites
B. Fleas
C. Ticks
D. Mosquitoes
Answer: C
Explanation:

(A) Mites are not the common vector of Lyme disease.
(B) Fleas are not the common vector of Lyme disease.
(C) Ticks are the common vector of Lyme disease.
(D) Mosquitoes are not the common vector of Lyme disease.
QUESTION 73
A child is admitted to the emergency room with her mother. Her mother states that she has
been exposed to chickenpox. During the assessment, the nurse would note a characteristic
rash:
A. That is covered with vesicular scabs all in the macular stage
B. That appears profusely on the trunk and sparsely on the extremities
C. That first appears on the neck and spreads downward
D. That appears especially on the cheeks, which gives a “slapped-cheek appearance”
Answer: B
Explanation:
(A) A rash with vesicular scabs in all stages (macule, papule, vesicle, and crusts).
(B) A rash that appears profusely on the trunk and sparsely on the extremities.
(C) A rash that first appears on the neck and spreads downward is characteristic of rubeola
and rubella.
(D) A rash, especially on the cheeks, that gives a “slapped-cheek appearance is characteristic
of roseola.”
QUESTION 74
A client is 6 weeks pregnant. During her first prenatal visit, she asks, “How much alcohol is
safe to drink during pregnancy?” The nurse’s response is:
A. Up to 1 oz daily
B. Up to 2 oz daily
C. Up to 4 oz weekly
D. No alcohol
Answer: D
Explanation:
(A, B, C) No amount of alcohol has been determined safe for pregnant women. Alcohol
should be avoided owing to the risk of fetal alcohol syndrome.
(D) The recommended safe dosage of alcohol consumption during pregnancy is none.

QUESTION 75
Which of the following would the nurse expect to find following respiratory assessment of a
client with advanced emphysema?
A. Distant breath sounds
B. Increased heart sounds
C. Decreased anteroposterior chest diameter
D. Collapsed neck veins
Answer: A
Explanation:
(A) Distant breath sounds are found in clients with emphysema owing to increased
anteroposterior chest diameter, overdistention, and air trapping.
(B) Deceased heart sounds are present because of the increased anteroposterior chest
diameter.
(C) A barrel-shaped chest is characteristic of emphysema.
(D) Increased distention of neck veins is found owing to right-sided heart failure, which may
be present in advanced emphysema.
QUESTION 76
Which of the following would have the physiological effect of decreasing intracranial
pressure (ICP)?
A. Increased core body temperature
B. Decreased serum osmolality
C. Administration of hypo-osmolar fluids
D. Decreased PaCO2
Answer: D
Explanation:
(A) An increase in core body temperature increases metabolism and results in an increase in
ICP.
(B) Decreased serum osmolality indicates a fluid overload and may result in an increase in
ICP.
(C) Hypo-osmolar fluids are generally voided in the neurologically compromised. Using IV
fluids such as D5W results in the dextrose being metabolized, releasing free water that is
absorbed by the brain cells, leading to cerebral edema.

(D) Hypercapnia and hypoventilation, which cause retention of CO2 and lead to respiratory
acidosis, both increase ICP. CO2 is the most potent vasodilator known.
QUESTION 77
When teaching a mother of a 4-month-old with diarrhea about the importance of preventing
dehydration, the nurse would inform the mother about the importance of feeding her child:
A. Fruit juices
B. Diluted carbonated drinks
C. Soy-based, lactose-free formula
D. Regular formulas mixed with electrolyte solutions
Answer: C
Explanation:
(A) Diluted fruit juices are not recommended for rehydration because they tend to aggravate
the diarrhea.
(B) Diluted soft drinks have a high-carbohydrate content, which aggravates the diarrhea.
(C) Soy-based, lactose-free formula reduces stool output and duration of diarrhea in most
infants.
(D) Regular formulas contain lactose, which can increase diarrhea.
QUESTION 78
The nurse is aware that nutrition is an important aspect of care for a client with hepatitis.
Which of the following diets would be most therapeutic?
A. High protein and low carbohydrate
B. Low calorie and low protein
C. High carbohydrate and high calorie
D. Low carbohydrate and high calorie
Answer: C
Explanation:
(A) Protein increases the workload of the liver. Increased carbohydrates provide needed
calories and promote palatability.
(B) Dietary intake should be adequate to ensure wound healing.
(C) Increased carbohydrates provide needed calories.
(D) A high calorie diet is best obtained from carbohydrates because of their palatability. Fats
increase the workload of the liver.

QUESTION 79
A long-term goal for the nurse in planning care for a depressed, suicidal client would be to:
A. Provide him with a safe and structured environment.
B. Assist him to develop more effective coping mechanisms.
C. Have him sign a “no-suicide contract.”
D. Isolate him from stressful situations that may precipitate a depressive episode.
Answer: B
Explanation:
(A) This statement represents a short-term goal.
(B) Long-term therapy should be directed toward assisting the client to cope effectively with
stress.
(C) Suicide contracts represent short-term interventions.
(D) This statement represents an unrealistic goal. Stressful situations cannot be avoided in
reality.
QUESTION 80
The primary reason that an increase in heart rate (100 bpm) detrimental to the client with a
myocardial infarction (MI) is that:
A. Stroke volume and blood pressure will drop proportionately
B. Systolic ejection time will decrease, thereby decreasing cardiac output
C. Decreased contractile strength will occur due to decreased filling time
D. Decreased coronary artery perfusion due to decreased diastolic filling time will occur,
which will increase ischemic damage to the myocardium
Answer: D
Explanation:
(A) Decreased stroke volume and blood pressure will occur secondary to decreased diastolic
filling.
(B) Tachycardia primarily decreases diastole; systolic time changes very little.
(C) Contractility decreases owing to the decreased filling time and decreased time for fiber
lengthening.
(D) Decreased O2 supply due to decreased time for filling of the coronary arteries increases
ischemia and infarct size. Tachycardia primarily robs the heart of diastolic time, which is the
primary time for coronary artery filling.

QUESTION 81
Which of the following nursing orders should be included in the plan of care for a client with
hepatitis C?
A. The nurse should use universal precautions when obtaining blood samples.
B. Total bed rest should be maintained until the client is asymptomatic.
C. The client should be instructed to maintain a low semi-Fowler position when eating meals.
D. The nurse should administer an alcohol backrub at bedtime.
Answer: A
Explanation:
(A) The source of infection with hepatitis C is contaminated blood products.
(B) Modified bed rest should be maintained while the client is symptomatic. Routine
activities can be slowly resumed once the client is asymptomatic.
(C) Nausea and vomiting occur frequently with hepatitis C. A high Fowler position may
decrease the tendency to vomit.
(D) The buildup of bilirubin in the client’s skin may cause pruritus. Alcohol is a drying agent.
QUESTION 82
When inspecting a cardiovascular client, the nurse notes that he needs to sit upright to
breathe. This behavior is most indicative of:
A. Pericarditis
B. Anxiety
C. Congestive heart failure
D. Angina
Answer: C
Explanation:
(A) Pericarditis can cause dyspnea but primarily causes chest pain.
(B) Anxiety can cause dyspnea resulting in SOB, yet it is not typically influenced by degree
of head elevation.
(C) The inability to oxygenate well without being upright is most indicative of congestive
heart failure, due to alveolar drowning.
(D) Angina causes primarily chest pain; any SOB associated with angina is not influenced by
body position.

QUESTION 83
The physician has ordered that a daily exercise program be instituted by a client with type I
diabetes following his discharge from the hospital. Discharge instructions about exercise
should include which of the following?
A. Exercise should be performed 30 minutes before meals.
B. A snack may be needed before and/or during exercise.
C. Hyperglycemia may occur 2–4 hours after exercise.
D. The blood glucose level should be 100 mg or below before exercise is begun.
Answer: B
Explanation:
(A) Exercise should not be performed before meals because the blood sugar is usually lower
just prior to eating; therefore, there is an increased risk for hypoglycemia.
(B) Exercise lowers blood sugar levels; therefore, a snack may be needed to maintain the
appropriate glucose level.
(C) Exercise lowers blood sugar levels.
(D) Exercise lowers blood sugar levels. If the blood glucose level is 100 mg or below at the
start of exercise, the potential for hypoglycemia is greater.
QUESTION 84
The nurse notes hyperventilation in a client with a thermal injury. She recognizes that this
may be a reaction to which of the following medications if applied in large amounts?
A. Neosporin sulfate
B. Mafenide acetate
C. Silver sulfadiazine
D. Povidone-iodine
Answer: B
Explanation:
(A) The side effects of neomycin sulfate include rash, urticaria, nephrotoxicity, and
ototoxicity.
(B) The side effects of mafenide acetate include bone marrow suppression, hemolytic anemia,
eosinophilia, and metabolic acidosis. The hyperventilation is a compensatory response to the
metabolic acidosis.
(C) The side effects of silver sulfadiazine include rash, itching, leukopenia, and decreased
renal function.

(D) The primary side effect of povidone- iodine is decreased renal function.
QUESTION 85
The nurse practitioner determines that a client is approximately 9 week’s gestation. During
the visit, the practitioner informs the client about symptoms of physical changes that she will
experience during her first trimester, such as:
A. Nausea and vomiting
B. Quickening
C. A 6–8 lb weight gain
D. Abdominal enlargement
Answer: A
Explanation:
(A) Nausea and vomiting are experienced by almost half of all pregnant women during the
first 3 months of pregnancy as a result of elevated human chorionic gonadotropin levels and
changed carbohydrate metabolism.
(B) Quickening is the mother’s perception of fetal movement and generally does not occur
until 18–20 weeks after the last menstrual period in primigravidas, but it may occur as early
as 16 weeks in multigravidas.
(C) During the first trimester there should be only a modest weight gain of 2–4 lb. It is not
uncommon for women to lose weight during the first trimester owing to nausea and/or
vomiting.
(D) Physical changes are not apparent until the second trimester, when the uterus rises out of
the pelvis.
QUESTION 86
Which of the following medications requires close observation for bronchospasm in the client
with chronic obstructive pulmonary disease or asthma?
A. Verapamil (Isoptin)
B. Amrinone (Inocor)
C. Epinephrine (Adrenalin)
D. Propranolol (Inderal)
Answer: D
Explanation:

(A) Verapamil has the respiratory side effect of nasal or chest congestion, dyspnea, shortness
of breath (SOB), and wheezing.
(B) Amrinone has the effect of increased contractility and dilation of the vascular smooth
muscle. It has no noted respiratory side effects.
(C) Epinephrine has the effect of bronchodilation through stimulation.
(D) Propranolol, esmolol, and labetalol are all - blocking agents, which can increase airway
resistance and cause bronchospasms.
QUESTION 87
Which of the following procedures is necessary to establish a definitive diagnosis of breast
cancer?
A. Diaphanography
B. Mammography
C. Thermography
D. Breast tissue biopsy
Answer: D
Explanation:
(A) Diaphanography, also known as transillumination, is a painless, non-invasive imaging
technique that involves shining a light source through the breast tissue to visualize the
interior. It must be used in conjunction with a mammogram and physical examination.
(B) Mammography is a useful tool for screening but is not considered a means of diagnosing
breast cancers.
(C) Thermography is a pictorial representation of heat patterns on the surface of the breast.
Breast cancers appear as a “hot spot owing to their higher metabolic rate.”
(D) Biopsy either by needle aspiration or by surgical incision is the primary diagnostic
technique for confirming the presence of cancer cells.
QUESTION 88
Pregnant women with diabetes often have problems related to the effectiveness of insulin in
controlling their glucose levels during their second half of pregnancy. The nurse teaches the
client that this is due to:
A. Decreased glomerular filtration and increased tubular absorption
B. Decreased estrogen levels
C. Decreased progesterone levels

D. Increased human placental lactogen levels
Answer: D
Explanation:
(A) There is a rise in glomerular filtration rate in the kidneys in conjunction with decreased
tubular glucose reabsorption, resulting in glycosuria.
(B) Insulin is inhibited by increased levels of estrogen.
(C) Insulin is inhibited by increased levels of progesterone.
(D) Human placental lactogen levels increase later in pregnancy. This hormonal antagonist
reduces insulin’s effectiveness, stimulates lipolysis, and increases the circulation of free fatty
acids.
QUESTION 89
A pregnant woman at 36 week’s gestation is followed for PIH and develops proteinuria. To
increase protein in her diet, which of the following foods will provide the greatest amount of
protein when added to her intake of 100 mL of milk?
A. Fifty milliliters light cream and 2 tbsp corn syrup
B. Thirty grams powdered skim milk and 1 egg
C. One small scoop (90 g) vanilla ice cream and 1 tbsp chocolate syrup
D. One package vitamin-fortified gelatin drink
Answer: B
Explanation:
(A) This choice would provide more unwanted fat and sugar than protein.
(B) Skim milk would add protein. Eggs are good sources of protein while low in fat and
calories.
(C) The benefit of protein from ice cream would be outweighed by the fat content. Chocolate
syrup has caffeine, which is contraindicated or limited in pregnancy.
(D) Although most animal proteins are higher in protein than plant proteins, gelatin is not. It
loses protein during the processing for food consumption.
QUESTION 90
A 3-year-old child is hospitalized with burns covering her trunk and lower extremities. Which
of the following would the nurse use to assess adequacy of fluid resuscitation in the burned
child?
A. Blood pressure

B. Serum potassium level
C. Urine output
D. Pulse rate
Answer: C
Explanation:
(A) Blood pressure can remain normotensive even in a state of hypovolemia.
(B) Serum potassium is not reliable for determining adequacy of fluid resuscitation.
(C) Urine output, alteration in sensorium, and capillary refill are the most reliable indicators
for assessing adequacy of fluid resuscitation.
(D) Pulse rate may vary for many reasons and is not a reliable indicator for assessing
adequacy of fluid resuscitation.
QUESTION 91
The medication that best penetrates eschar is:
A. Mafenide acetate (Sulfamylon)
B. Silver sulfadiazine (Silvadene)
C. Neomycin sulfate (Neosporin)
D. Povidone-iodine (Betadine)
Answer: A
Explanation:
(A) Mafenide acetate is bacteriostatic against gram-positive and gram-negative organisms and
is the agent that best penetrates eschar.
(B) Silver sulfadiazine poorly penetrates eschar.
(C) Neomycin sulfate does not penetrate eschar.
(D) Povidone iodine does not penetrate eschar.
QUESTION 92
A 25-year-old client believes she may be pregnant with her first child. She schedules an
obstetric examination with the nurse practitioner to determine the status of her possible
pregnancy. Her last menstrual period began May 20, and her estimated date of confinement
using Nägele’s rule is:
A. March 27
B. February 1
C. February 27

D. January 3
Answer: C
Explanation:
(A) March 27 is a miscalculation.
(B) February 1 is a miscalculation.
(C) February 27 is the correct answer. To calculate the estimated date of confinement using
Nagele’s rule, subtract 3 months from the date that the last menstrual cycle began and then
add 7 days to the result.
(D) January 3 is a miscalculation.
QUESTION 93
The therapeutic blood-level range for lithium is:
A. 0.25–1.0 mEq/L
B. 0.5–1.5 mEq/L
C. 1.0–2.0 mEq/L
D. 2.0–2.5 mEq/L
Answer: B
Explanation:
(A) This range is too low to be therapeutic.
(B) This is the therapeutic range for lithium.
(C) This range is above the therapeutic level.
(D) This range is toxic and may cause severe side effects.
QUESTION 94
Which of the following ECG changes would be seen as a positive myocardial stress test
response?
A. Hyperacute T wave
B. Prolongation of the PR interval
C. ST-segment depression
D. Pathological Q wave
Answer: C
Explanation:
(A) Hyperacute T waves occur with hyperkalemia.
(B) Prolongation of the P R interval occurs with first-degree AV block.

(C) Horizontal ST-segment depression of 1 mm during exercise is definitely a positive
criterion on the exercise ECG test.
(D) Patho-logical Q waves occur with MI.
QUESTION 95
A psychotic client who believes that he is God and rules all the universe is experiencing
which type of delusion?
A. Somatic
B. Grandiose
C. Persecutory
D. Nihilistic
Answer: B
Explanation:
(A) These delusions are related to the belief that an individual has an incurable illness.
(B) These delusions are related to feelings of self-importance and uniqueness.
(C) These delusions are related to feelings of being conspired against.
(D) These delusions are related to denial of self-existence.
QUESTION 96
During burn therapy, morphine is primarily administered IV for pain management because
this route:
A. Delays absorption to provide continuous pain relief
B. Facilitates absorption because absorption from muscles is not dependable
C. Allows for discontinuance of the medication if respiratory depression develops
D. Avoids causing additional pain from IM injections
Answer: B
Explanation:
(A) Absorption would be increased, not decreased.
(B) IM injections should not be used until the client is hemodynamically stable and has
adequate tissue perfusion. Medications will remain in the subcutaneous tissue with the fluid
that is present in the interstitial spaces in the acute phase of the thermal injury. The client will
have a poor response to the medication administered, and a “dumping” of the medication can
occur when the medication and fluid are shifted back into the intravascular spaces in the next
phase of healing.

(C) IV administration of the medication would hasten respiratory compromise, if present.
(D) The desire to avoid causing the client additional pain is not a primary reason for this route
of administration.
QUESTION 97
Assessment of the client with pericarditis may reveal which of the following?
A. Ventricular gallop and substernal chest pain
B. Narrowed pulse pressure and shortness of breath
C. Pericardial friction rub and pain on deep inspiration
D. Pericardial tamponade and widened pulse pressure
Answer: C
Explanation:
(A) No S3 or S4 are noted with pericarditis.
(B) No change in pulse pressure occurs.
(C) The symptoms of pericarditis vary with the cause, but they usually include chest pain,
dyspnea, tachycardia, rise in temperature, and friction rub caused by fibrin or other deposits.
The pain seen with pericarditis typically worsens with deep inspiration.
(D) Tamponade is not typically seen early on, and no change in pulse pressure occurs.
QUESTION 98
A client who has sustained a basilar skull fracture exhibits blood-tinged drainage from his
nose. After establishing a clear airway, administering supplemental O2, and establishing IV
access, the next nursing intervention would be to:
A. Pass a nasogastric tube through the left nostril
B. Place a 4 X 4 gauze in the nares to impede the flow
C. Gently suction the nasal drainage to protect the airway
D. Perform a halo test and glucose level on the drainage
Answer: D
Explanation:
(A) Basilar skull fracture may cause dural lacerations, which result in CSF leaking from the
ears or nose. Insertion of a tube could lead to CSF going into the brain tissue or sinuses.
(B) Tampon ding flow could worsen the problem and increase ICP.
(C) Suction could increase brain damage and dislocate tissue.

(D) Testing the fluid from the nares would determine the presence of CSF. Elevation of the
head, notification of the medical staff, and prophylactic antibiotics are appropriate therapy.
QUESTION 99
To appropriately monitor therapy and client progress, the nurse should be aware that
increased myocardial work and O2 demand will occur with which of the following?
A. Positive inotropic therapy
B. Negative chronotropic therapy
C. Increase in balance of myocardial O2 supply and demand
D. Afterload reduction therapy
Answer: A
Explanation:
(A) Inotropic therapy will increase contractility, which will increase myocardial O2 demand.
(B) Decreased heart rate to the point of bradycardia will increase coronary artery filling time.
This should be used cautiously because tachycardia may be a compensatory mechanism to
increase cardiac output.
(C) The goal in the care of the MI client with angina is to maintain a balance between
myocardial O2 supply and demand.
(D) Decrease in systemic vascular resistance by drug therapy, such as IV nitroglycerin or
nitroprusside, or intra-aortic balloon pump therapy, would decrease myocardial work and O2
demand.
QUESTION 100
The most important reason to closely assess circumferential burns at least every hour is that
they may result in:
A. Hypovolemia
B. Renal damage
C. Ventricular arrhythmias
D. Loss of peripheral pulses
Answer: D
Explanation:
(A) Hypovolemia could be a result of fluid loss from thermal injury, but not as a result of the
circumferential injury.
(B) Renal damage is typically seen because of prolonged hypovolemia or myoglobinuria.

(C) Electrical injuries and electrolyte changes typically cause arrhythmias in the burn client.
(D) Full- thickness circumferential burns are nonelastic and result in an internal tourniquet
effect that compromises distal blood flow when the area involved is an extremity.
Circumferential full-thickness torso burns compromise respiratory motion and, when
extreme, cardiac return.
QUESTION 101
A pregnant client comes to the office for her first prenatal examination at 10 weeks. She has
been pregnant twice before; the first delivery produced a viable baby girl at 39 weeks 3 years
ago; the second pregnancy produced a viable baby boy at 36 weeks 2 years ago. Both
children are living and well. Using the GTPAL system to record her obstetrical history, the
nurse should record:
A. 3-2-0-0-2
B. 2-2-0-2-2
C. 3-1-1-0-2
D. 2-1-1-0-2
Answer: C
Explanation:
(A) This answer is an incorrect application of the GTPAL method. One prior pregnancy was a
preterm birth at 36 weeks (T = 1, P = 1; not T = 2).
(B) This answer is an incorrect application of the GTPAL method. The client is currently
pregnant for the third time (G = 3, not 2), one prior pregnancy was preterm (T = 1, P = 1; not
T = 2), and she has had no prior abortions (A = 0).
(C) This answer is the correct application of GTPAL method. The client is currently pregnant
for the third time (G = 3), her first pregnancy ended at term (37 weeks) (T = 1), her second
pregnancy ended preterm 20–33 weeks) (P = 1), she has no history of abortion (A = 0), and
she has two living children (L = 2).
(D) This answer is an incorrect application of the GTPAL method. The client is currently
pregnant for the third time (G = 3, not 2).
QUESTION 102
A 68-year-old woman is admitted to the hospital with chronic obstructive pulmonary disease
(COPD). She is started on an aminophylline infusion. Three days later she is breathing easier.

A serum theophylline level is drawn. Which of the following values represents a therapeutic
level?
A. 14 µ g/mL
B. 25 µ g/mL
C. 4 µ g/mL
D. 30 µ g/mL
Answer: A
Explanation:
(A) The therapeutic blood level range of theophylline is 10–20 mg/mL. Therapeutic drug
monitoring determines effective drug dosages and prevents toxicity.
(B, D) This value is a toxic level of the drug.
(C) This value is a nontherapeutic level of the drug.
QUESTION 103
A nasogastric (NG) tube inserted preoperatively is attached to low, intermittent suctions. A
client with an NG tube exhibits these symptoms: He is restless; serum electrolytes are Na
138, K 4.0, blood pH 7.53. This client is most likely experiencing:
A. Hyperkalemia
B. Hyponatremia
C. Metabolic acidosis
D. Metabolic alkalosis
Answer: D
Explanation:
(A) Sodium level is within normal limits.
(B) Sodium level is within normal limits.
(C) pH level is consistent with alkalosis.
(D) With an NG tube attached to low, intermittent suction, acids are removed and a client will
develop metabolic alkalosis.
QUESTION 104
A client is in early labor. Her fetus is in a left occipitoanterior (LOA) position; fetal heart
sounds are best auscultated just:
A. Below the umbilicus toward left side of mother’s abdomen
B. Below the umbilicus toward right side of mother’s abdomen

C. At the umbilicus
D. Above the umbilicus to the left side of mother’s abdomen
Answer: A
Explanation:
(A) LOA identifies a fetus whose back is on its mother’s left side, whose head is the
presenting part, and whose back is toward its mother’s anterior. It is easiest to auscultate fetal
heart tones (FHTs) through the fetus back.
(B) The identified fetus back is on its mother’s left side, not right side. It is easiest to
auscultate FHTs through the fetus back.
(C) In an LOA position, the fetus head is presenting with the back to the left anterior side of
the mother. The umbilicus is too high of a landmark for auscultating the fetus heart rate
through its back.
(D) This is the correct auscultation point for a fetus in the left Sacro anterior position, where
the sacrum is presenting, not LOA.
QUESTION 105
A mother came to the pediatric clinic with her 17- month-old child. The mother would like to
begin toilet training. What should the nurse teach her about implementing toilet training?
A. Take two or three favorite toys with the child.
B. Have a child-sized toilet seat or training potty on hand.
C. Explain to the child she is going to “void” and “defecate”.
D. Show disapproval if she does not void or defecate.
Answer: B
Explanation:
(A) Giving her toys will distract her and interfere with toilet training because of inappropriate
reinforcement.
(B) A child-sized toilet seat or training potty gives a child a feeling of security.
(C) She should use words that are age appropriate for the child.
(D) Children should be praised for cooperative behavior and/or successful evacuation.
QUESTION 106
Which one of the following is considered a reliable indicator for assessing the adequacy of
fluid resuscitation in a 3-year-old child who suffered partial- and full thickness burns to 25%
of her body?

A. Urine output
B. Edema
C. Hypertension
D. Bulging fontanelle
Answer: A
Explanation:
(A) Urinary output is a reliable indicator of renal perfusion, which in turn indicates that fluid
resuscitation is adequate. IV fluids are adjusted based on the urinary output of the child
during fluid resuscitation.
(B) Edema is an indication of increased capillary permeability following a burn injury.
(C) Hypertension is an indicator of fluid volume excess.
(D) Fontanelles close by 18 months of age.
QUESTION 107
The nurse working in a prenatal clinic needs to be alert to the cardinal signs and symptoms of
PIH because:
A. Immediate treatment of mild PIH includes the administration of a variety of medications
B. Psychological counseling is indicated to reduce the emotional stress causing the blood
pressure elevation
C. Self-discipline is required to control caloric intake throughout the pregnancy
D. The client may not recognize the early symptoms of PIH
Answer: D
Explanation:
(A) Mild PIH is not treated with medications.
(B) Emotional stress is not the cause of blood pressure elevation in PIH.
(C) Excessive caloric intake is not the cause of weight gain in PIH.
(D) The client most frequently is not aware of the signs and symptoms in mild PIH.
QUESTION 108
A client is admitted to the labor unit. On vaginal examination, the presenting part in a
cephalic presentation was at station plus two. Station 12 means that the:
A. Presenting part is 2 cm above the level of the ischial spines
B. Biparietal diameter is at the level of the ischial spines
C. Presenting part is 2 cm below the level of the ischial spines

D. Biparietal diameter is 5 cm above the ischial spines
Answer: C
Explanation:
(A) Station is the relationship of the presenting part to an imaginary line drawn between the
ischial spines. If the presenting part is above the ischial spines, the station is negative.
(B) When the biparietal diameter is at the level of the ischial spines, the presenting part is
generally at a +4 or +5 station.
(C) Station is the relationship of the presenting part to an imaginary line drawn between the
ischial spines. If the presenting part is below the ischial spines, the station is positive. Thus, 2
cm below the ischial spines is the station +2.
(D) When the biparietal diameter is above the ischial spines by 5 cm, the presenting part is
usually engaged or at station 0.
QUESTION 109
To prevent fungal infections of the mouth and throat, the nurse should teach clients on inhaled
steroids to:
A. Rinse the plastic holder that aerosolizes the drug with hydrogen peroxide every other day
B. Rinse the mouth and gargle with warm water after each use of the inhaler
C. Take antacids immediately before inhalation to neutralize mucous membranes and prevent
infection
D. Rinse the mouth before each use to eliminate colonization of bacteria
Answer: B
Explanation:
(A) It is sufficient to rinse the plastic holders with warm water at least once per day.
(B) It is important to rinse the mouth after each use to minimize the risk of fungal infections
by reducing the droplets of the glucocorticoid left in the oral cavity.
(C) Antacids act by neutralizing or reducing gastric acid, thus decreasing the pH of the
stomach. “Neutralizing” the oral mucosa prior to inhalation of a steroid inhaler does not
minimize the risk of fungal infections.
(D) Rinsing prior to the use of the glucocorticoid will not eliminate the droplets left on the
oral mucous membranes following the use of the inhaler.

QUESTION 110
A 16-year-old client comes to the prenatal clinic for her monthly appointment. She has gained
14 lb from her 7th to 8th month; her face and hands indicate edema. She is diagnosed as
having PIH and referred to the high-risk prenatal clinic. The client’s weight increase is most
likely due to:
A. Overeating and subsequent obesity
B. Obesity prior to conception
C. Hypertension due to kidney lesions
D. Fluid retention
Answer: D
Explanation:
(A) Overeating can lead to obesity, but not to edema.
(B) There is no indication of obesity prior to pregnancy. PIH is more prevalent in the
underweight than in the obese in this age group.
(C) Hypertension can be due to kidney lesions, but it would have been apparent earlier in the
pregnancy.
(D) The weight gain in PIH is due to the retention of sodium ions and fluid and is one of the
three cardinal symptoms of PIH.
QUESTION 111
A 6-month-old infant has developmental delays. His weight falls below the 5th percentile
when plotted on a growth chart. A diagnosis of failure to thrive is made. What behaviors
might indicate the possibility of maternal deprivation?
A. Responsive to touch, wants to be held
B. Uncomforted by touch, refuses bottle
C. Maintains eye-to-eye contact
D. Finicky eater, easily pacified, cuddly
Answer: B
Explanation:
(A) Normal infant attachment behaviors include responding to touch and wanting to be held.
(B) Maternal deprivation behaviors include poor feeding, stiffening and refusal to eat, and
inconsistencies in responsiveness.
(C) Attachment behavior includes maintaining eye contact.
(D) Maternal deprivation behaviors include displeasure with touch and physical contact.

QUESTION 112
The nurse teaches a male client ways to reduce the risks associated with furosemide therapy.
Which of the following indicates that he understands this teaching?
A. “I’ll be sure to rise slowly and sit for a few minutes after lying down.”
B. “I’ll be sure to walk at least 2–3 blocks every day.”
C. “I’ll be sure to restrict my fluid intake to four or five glasses a day.”
D. “I’ll be sure not to take any more aspirin while I am on this drug.”
Answer: A
Explanation:
(A) This response will help to prevent the occurrence of postural hypotension, a common side
effect of this drug and a common reason for falls.
(B) Although walking is an excellent exercise, it is not specific to the reduction of risks
associated with diuretic therapy.
(C) Clients on diuretic therapy are generally taught to ensure that their fluid intake is at least
2000–3000 mL daily, unless contraindicated.
(D) Aspirin is a safe drug to take along with furosemide.
QUESTION 113
A client is admitted to the hospital for an induction of labor owing to a gestation of 42 weeks
confirmed by dates and ultrasound. When she is dilated 3 cm, she has a contraction of 70
seconds. She is receiving oxytocin. The nurse’s first intervention should be to:
A. Check FHT
B. Notify the attending physician
C. Turn off the IV oxytocin
D. Prepare for the delivery because the client is probably in transition
Answer: C
Explanation:
(A) FHT should be monitored continuously with an induction of labor; this is an accepted
standard of care.
(B) The physician should be notified, but this is not the first intervention the nurse should do.
(C) The standard of care for an induction according to the Association of Women’s Health,
Obstetric, and Neonatal Nurses and American College of Obstetrics and Gynecology is that
contractions should not exceed 60 seconds in an induction. Inductions should simulate normal

labor; 70-second contractions during the latent phase (3 cm) are not the norm. The next
contractions can be longer and increase risks to the mother and fetus.
(D) Contractions lasting 60–90 seconds during transition are typical; this provides a good
distractor. The nurse needs to be knowledgeable of the phases and stages of labor.
QUESTION 114
A male client is scheduled for a liver biopsy. In preparing him for this test, the nurse should:
A. Explain that he will be kept NPO for 24 hours before the exam
B. Practice with him so he will be able to hold his breath for 1 minute
C. Explain that he will be receiving a laxative to prevent a distended bowel from applying
pressure on the liver
D. Explain that his vital signs will be checked frequently after the test
Answer: D
Explanation:
(A) There is no NPO restriction prior to a liver biopsy.
(B) The client would need to hold his breath for 5–10 seconds.
(C) There is no pretest laxative given.
(D) Following the test, the client is watched for hemorrhage and shock.
QUESTION 115
After a liver biopsy, the best position for the client is:
A. High Fowler
B. Prone
C. Supine
D. Right lateral
Answer: D
Explanation:
(A) This position does not help to prevent bleeding.
(B) This position does not help to prevent bleeding.
(C) This position does not help to prevent bleeding.
(D) The right lateral position would allow pressure on the liver to prevent bleeding.

QUESTION 116
In assisting preconceptual clients, the nurse should teach that the corpus luteum secretes
progesterone, which thickens the endometrial lining in which of the phases of the menstrual
cycle?
A. Menstrual phase
B. Proliferative phase
C. Secretory phase
D. Ischemic phase
Answer: C
Explanation:
(A) Menses occurs during the menstrual phase, during which levels of both estrogen and
progesterone are decreased.
(B) The ovarian hormone responsible for the proliferative phase, during which the uterine
endometrium enlarges, is estrogen.
(C) The ovarian hormone responsible for the secretory phase is progesterone, which is
secreted by the corpus luteum and causes marked swelling in the uterine endometrium.
(D) The corpus luteum begins to degenerate in the ischemic phase, causing a fall in both
estrogen and progesterone.
QUESTION 117
A 4-year-old child is being discharged from the hospital after being treated for severe croup.
Which one of the following instructions should the nurse give to the child’s mother for the
home treatment of croup?
A. Take him in the bathroom, turn on the hot water, and close the door.
B. Give him a dose of antihistamine.
C. Give large amounts of clear liquids if drooling occurs.
D. Place him near a cool mist vaporizer and encourage crying.
Answer: A
Explanation:
(A) Initial home treatment of croup includes placing the child in an environment of high
humidity to liquefy and mobilize secretions.
(B) Antihistamines should be avoided because they can cause thickening of secretions.

(C) Drooling is a characteristic sign of airway obstruction and the child should be taken
directly to the emergency room.
(D) Crying increases respiratory distress and hypoxia in the child with croup. The nurse
should promote methods that will calm the child.
QUESTION 118
A male client is started on IV anticoagulant therapy with heparin. Which of the following
laboratory studies will be ordered to monitor the therapeutic effects of heparin?
A. Partial thromboplastin time
B. Hemoglobin
C. Red blood cell (RBC) count
D. Prothrombin time
Answer: A
Explanation:
(A) Partial thromboplastin time is used to monitor the effects of heparin, and dosage is
adjusted depending on test results. It is a screening test used to detect deficiencies in all
plasma clotting factors except factors VII and XIII and platelets.
(B) Hemoglobin is the main component of RBCs. Its main function is to carry O2 from the
lungs to the body tissues and to transport CO2 back to the lungs.
(C) RBC count is the determination of the number of RBCs found in each cubic millimeter of
whole blood.
(D) PT is used to monitor the effects of oral anticoagulants, e.g., coumarin type
anticoagulants.
QUESTION 119
A 16-year-old student has a long history of bronchial asthma and has experienced several
severe asthmatic attacks during the school year. The school nurse is required to administer 0.2
mL of 1/1000 solution of epinephrine SC during an asthma attack. How does the school nurse
evaluate the effectiveness of this intervention?
A. Increased pulse rate
B. Increased expectorate of secretions
C. Decreased inspiratory difficulty
D. Increased respiratory rate
Answer: C

Explanation:
(A) A side effect of epinephrine is fatal ventricular fibrillation owing to its effects on cardiac
stimulation.
(B) Medications used to treat asthma are designed to decrease bronchospasm, not to increase
expectorate of secretions.
(C) Epinephrine decreased inspiratory difficulty by stimulating -, 1, and 2-receptors causing
sympathomimetic stimulation (e.g., bronchodilation).
(D) The person with asthma fights to inspire sufficient air thus increasing respiratory rate.
QUESTION 120
Nursing interventions designed to decrease the risk of infection in a client with an indwelling
catheter include:
A. Cleanse area around the meatus twice a day
B. Empty the catheter drainage bag at least daily
C. Change the catheter tubing and bag every 48 hours
D. Maintain fluid intake of 1200–1500 mL every day
Answer: A
Explanation:
(A) Catheter site care is to be done at least twice daily to prevent pathogen growth at the
catheter insertion site.
(B) Catheter drainage bags are usually emptied every 8 hours to prevent urine stasis and
pathogen growth.
(C) Tubing and collection bags are not changed this often, because research studies have not
demonstrated the efficacy of this practice.
(D) Fluid intake needs to be in the 2000–2500 mL range if possible to help irrigate the
bladder and prevent infection.
QUESTION 121
A 23-year-old borderline client is admitted to an inpatient psychiatric unit following an
impulsive act of self- mutilation. A few hours after admission, she requests special privileges,
and when these are not granted, she stands up and angrily shouts that the people on the unit
do not care, and she storms across the room. The nurse should respond to this behavior by:
A. Placing her in seclusion until the behavior is under control
B. Walking up to the client and touching her on the arm to get her attention

C. Communicating a desire to assist the client to regain control, offering a one-to-one session
in a quiet area
D. Confronting the client, letting her know the consequences for getting angry and disrupting
the unit
Answer: C
Explanation:
(A) Threatening a client with punitive action is violating a client’s rights and could escalate
the client’s anger.
(B) Angry clients need respect for personal space, and physical contact may be perceived as a
threatening gesture escalating anger.
(C) Client lacks sufficient self-control to limit own maladaptive behavior; she may need
assistance from staff.
(D) Confronting an angry client may escalate her anger to further acting out, and
consequences are for acting out anger aggressively, not for getting angry or feeling angry.
QUESTION 122
A client is being discharged with albuterol (Proventil) and beclomethasone dipropionate
(Vanceril) to be administered via inhalation three times a day and at bedtime. Client teaching
regarding the sequential order in which the drugs should be administered includes:
A. Glucocorticoid followed by the bronchodilator
B. Bronchodilator followed by the glucocorticoid
C. Alternate successive administrations
D. According to the client’s preference
Answer: B
Explanation:
(A) The client would not receive therapeutic effects of the glucocorticoid when it is inhaled
through constricted airways.
(B) Broncho dilating the airways first allows for the glucocorticoid to be inhaled through
open airways and increases the penetration of the steroid for maximum effectiveness of the
drug.
(C) Inac- Inaccurate use of the inhalers will lead to decreased effectiveness of the treatment.
(D) Client teaching regarding the use and effects of inhalers will promote client
understanding and compliance.

QUESTION 123
A gravida 2 para 1 client delivered a full-term newborn 12 hours ago. The nurse finds her
uterus to be boggy, high, and deviated to the right. The most appropriate nursing action is to:
A. Notify the physician
B. Place the client on a pad count
C. Massage the uterus and re-evaluate in 30 minutes
D. Have the client void and then re-evaluate the fundus
Answer: D
Explanation:
(A) The nurse should initiate actions to remove the most frequent cause of uterine
displacement, which involves emptying the bladder. Notifying the physician is an
inappropriate nursing action.
(B) The pad count gives an estimate of blood loss, which is likely to increase with a boggy
uterus; but this action does not remove the most frequent cause of uterine displacement,
which is a full bladder.
(C) Massage may firm the uterus temporarily, but if a full bladder is not emptied, the uterus
will remain displaced and is likely to relax again.
(D) The most common cause of uterine displacement is a full bladder.
QUESTION 124
During a client’s first postpartum day, the nurse assessed that the fundus was located laterally
to the umbilicus. This may be due to:
A. Endometritis
B. Fibroid tumor on the uterus
C. Displacement due to bowel distention
D. Urine retention or a distended bladder
Answer: D
Explanation:
(A, B) Endometritis, urine retention, or bladder distention provide good distractors because
they may delay involution but do not usually cause the uterus to be lateral.
(C) Bowel distention and constipation are common in the postpartum period but do not
displace the uterus laterally.
(D) Urine retention or bladder distention commonly displaces the uterus to the right and may
delay involution.

QUESTION 125
The nurse instructs a pregnant client (G2P1) to rest in a side-lying position and avoid lying
flat on her back. The nurse explains that this is to avoid “vena caval syndrome”, a condition
which:
A. Occurs when blood pressure increases sharply with changes in position
B. Results when blood flow from the extremities is blocked or slowed
C. Is seen mainly in first pregnancies
D. May require medication if positioning does not help
Answer: B
Explanation:
(A) Blood pressure changes are predominantly due to pressure of the gravid uterus.
(B) Pressure of the gravid uterus on the inferior vena cava decreases blood return from lower
extremities.
(C) Inferior vena cava syndrome is experienced in the latter months of pregnancy regardless
of parity.
(D) There are no medications useful in the treatment of interior vena cava syndrome;
alleviating pressure by position changes is effective.
QUESTION 126
The nurse enters the playroom and finds an 8-year-old child having a grand mal seizure.
Which one of the following actions should the nurse take?
A. Place a tongue blade in the child’s mouth.
B. Restrain the child so he will not injure himself.
C. Go to the nurses station and call the physician.
D. Move furniture out of the way and place a blanket under his head.
Answer: D
Explanation:
(A) The nurse should not put anything in the child’s mouth during a seizure; this action could
obstruct the airway.
(B) Restraining the child’s movements could cause constrictive injury.
(C) Staying with the child during a seizure provides protection and allows the nurse to
observe the seizure activity.
(D) The nurse should provide safety for the child by moving objects and protecting the head.

QUESTION 127
A mother frantically calls the emergency room (ER) asking what to do about her 3-year-old
girl who was found eating pills out of a bottle in the medicine cabinet. The ER nurse tells the
mother to:
A. Give the child 15 mL of syrup of ipecac.
B. Give the child 10 mL of syrup of ipecac with a sip of water.
C. Give the child 1 cup of water to induce vomiting.
D. Bring the child to the ER immediately.
Answer: D
Explanation:
(A) Before giving any emetic, the substance ingested must be known.
(B) At least 8 oz of water should be administered along with ipecac syrup to increase volume
in the stomach and facilitate vomiting.
(C) Water alone will not induce vomiting. An emetic is necessary to facilitate vomiting.
(D) Vomiting should never be induced in an unconscious client because of the risk of
aspiration.
QUESTION 128
A mother continues to breast-feed her 3-month-old infant. She tells the nurse that over the
past 3 days she has not been producing enough milk to satisfy the infant. The nurse advises
the mother to do which of the following?
A. “Start the child on solid food.”
B. “Nurse the child more frequently during this growth spurt.”
C. “Provide supplements for the child between breastfeeding so you will have enough milk.”
D. “Wait 4 hours between feedings so that your breasts will fill up.”
Answer: B
Explanation:
(A) Solid foods introduced before 4–6 months of age are not compatible with the abilities of
the GI tract and the nutritional needs of the infant.
(B) Production of milk is supply and demand. A common growth spurt occurs at 3 months of
age, and more frequent nursing will increase the milk supply to satisfy the infant.
(C) Supplementation will decrease the infant’s appetite and in turn decrease the milk supply.
When the infant nurses less often or with less vigor, the amount of milk produced decreases.

(D) Rigid feeding schedules lead to a decreased milk supply, whereas frequent nursing signals
the mother’s body to produce a correspondingly increased amount of milk.
QUESTION 129
A client is being discharged and will continue enteral feedings at home. Which of the
following statements by a family member indicates the need for further teaching?
A. “If he develops diarrhea lasting for more than 2–3 days, I will contact the doctor or nurse.”
B. “I should anticipate that he will gain about 1 lb/day now that he is on continuous
feedings.”
C. “It is important to keep the head of his bed elevated or sit him in the chair during
feedings.”
D. “I should use prepared or open formula within 24 hours and store unused portions in the
refrigerator.”
Answer: B
Explanation:
(A) Diarrhea is a complication of tube feedings that can lead to dehydration. Diarrhea may be
the result of hypertonic formulas that can draw fluid into the bowel. Other causes of diarrhea
may be bacterial contamination, fecal impaction, medications, and low albumin.
(B) A consistent weight gain of more than 0.22 kg/day (12 lb/day) over several days should
be reported promptly. The client should be evaluated for fluid volume excess.
(C) Elevating the client’s head prevents reflux and thus formula from entering the airway.
(D) Bacteria proliferate rapidly in enteral formulas and can cause gastroenteritis and even
sepsis.
QUESTION 130
Which nursing implication is appropriate for a client undergoing a paracentesis?
A. Have the client void before the procedure.
B. Keep the client NPO.
C. Observe the client for hypertension following the procedure.
D. Place the client on the right side following the procedure.
Answer: A
Explanation:
(A) A full bladder would impede withdrawal of ascitic fluid.
(B) Keeping the client NPO is not necessary.

(C) The client may exhibit signs and symptoms of shock and hypertension.
(D) No position change is needed after the procedure.
QUESTION 131
One of the medications that is prescribed for a male client is furosemide (Lasix) 80 mg bid.
To reduce his risk of falls, the nurse would teach him to take this medication:
A. On arising and no later than 6 PM
B. At evenly spaced intervals, such as 8 AM and 8 PM
C. With at least one glass of water per pill
D. With breakfast and at bedtime
Answer: A
Explanation:
(A) This option provides adequate spacing of the medication and will limit the client’s need to
get up to go to the bathroom during the night hours, when he is especially at high risk for
falls.
(B) This option would result in the need to get up during the night to urinate and would thus
increase the risk of falls. This option also does not take into consideration the client’s usual
daily routine.
(C) Taking this medication with at least one glass of water would not have an impact on the
risk of falls.
(D) This option would result in the need to get up during the night to urinate and would thus
increase the risk of falls.
QUESTION 132
The nurse observes that a client has difficulty chewing and swallowing her food. A nursing
response designed to reduce this problem would include:
A. Ordering a full liquid diet for her
B. Ordering five small meals for her
C. Ordering a mechanical soft diet for her
D. Ordering a puréed diet for her
Answer: C
Explanation:

(A) Full liquids would be difficult to swallow if the muscle control of the swallowing act is
affected; this is a probable reason for her difficulties, given her medical diagnosis of multiple
sclerosis.
(B) Five small meals would do little if anything to decrease her swallowing difficulties, other
than assure that she tires less easily.
(C) A mechanical soft diet should be easier to chew and swallow, because foods would be
more evenly consistent.
(D) A pureed diet would cause her to regress more than might be needed; the mechanical soft
diet should be tried first.
QUESTION 133
In performing the initial nursing assessment on a client at the prenatal clinic, the nurse will
know that which of the following alterations is abnormal during pregnancy?
A. Striae gravidarum
B. Chloasma
C. Dysuria
D. Colostrum
Answer: C
Explanation:
(A) Striae gravidarum are the normal stretch marks that frequently occur on the breasts,
abdomen, and thighs as pregnancy progresses.
(B) Chloasma is the “mask of pregnancy” that normally occurs in many pregnant women.
(C) Dysuria is an abnormal danger sign during pregnancy and may indicate a urinary tract
infection.
(D) Colostrum is a yellow breast secretion that is normally present during the last trimester of
pregnancy.
QUESTION 134
Assessment of severe depression in a client reveals feelings of hopelessness, worthlessness;
inability to feel pleasure; sleep, psychomotor, and nutritional alterations; delusional thinking;
negative view of self; and feelings of abandonment. These clinical features of the client’s
depression alert the nurse to prioritize problems and care by addressing which of the
following problems first:
A. Nutritional status

B. Impaired thinking
C. Possible harm to self
D. Rest and activity impairment
Answer: C
Explanation:
(A) Anorexia and weight loss are problems that need attention in severe depression, but they
can be addressed secondary to immediate concerns.
(B) Impaired thinking and confusion are problems in severe depression that are addressed
with administration of medication, through group and individual psychotherapy, and through
activity therapy as motivation and interest increase.
(C) Possible harm to self as with suicidal ideation; a suicide plan, means to execute plan;
and/or overt gestures or an attempt must be addressed as an immediate concern and safety
measures implemented appropriate to the risk of suicide.
(D) Rest and activity impairment may take time and further assessment to determine client’s
sleep pattern and amount of psychomotor retardation with the more immediate concern for
safety present.
QUESTION 135
The nurse is teaching a mother care of her child’s spica cast. The mother states that he
complains of itching under the edge of the cast. One nonpharmacological technique the nurse
might suggest would be:
A. “Blowing air under the cast using a hair dryer on cool setting often relieves itching.”
B. “Slide a ruler under the cast and scratch the area.”
C. “Guide a towel under and through the cast and move it back and forth to relieve the itch.”
D. “Gently thump on cast to dislodge dried skin that causes the itching.”
Answer: A
Explanation:
(A) Cool air will often relieve pruritus without damaging the cast or irritating the skin.
(B) The nurse should never force anything under the cast, because the cast may become
damaged and skin breakdown may occur.
(C) Forcing an object under the cast could lead to cast damage and skin breakdown. The
object may become lodged under the cast necessitating cast removal.
(D) This technique does not dislodge skin cells. It could damage the cast and cause skin
breakdown.

QUESTION 136
A 56-year-old psychiatric inpatient has had recurring episodes of depression and chronic low
self-esteem. She feels that her family does not want her around, experiences a sense of
helplessness, and has a negative view of herself. To assist the client in focusing on her
strengths and positive traits, a strategy used by the nurse would be to:
A. Tell the client to attend all structured activities on the unit
B. Encourage or direct client to attend activities that offer simple methods to attain success
C. Increase the client’s self-esteem by asking that she make all decisions regarding attendance
in group activities
D. Not allow any dependent behaviors by the client because she must learn independence and
will have to ask for any assistance from staff
Answer: B
Explanation:
(A) The nurse should encourage activities gradually, as client’s energy level and tolerance for
shared activities improve.
(B) Activities that focus on strengths and accomplishments, with uncomplicated tasks,
minimize failure and increase self-worth.
(C) Asking a client to set a goal to make all decisions about attending group activities is
unrealistic, and such decisions are not always under the client’s control; this sets up the client
for further failure and possibly decreased self-worth.
(D) Encouragement toward independence does promote increased feelings of self-worth;
however, clients may need assistance with decision making and problem solving for various
situations and on an individual basis.
QUESTION 137
Respiratory function is altered in a 16-year-old asthmatic. Which of the following is the cause
of this alteration?
A. Altered surfactant production
B. Paradoxical movements of the chest wall
C. Increased airway resistance
D. Continuous changes in respiratory rate and depth
Answer: C
Explanation:

(A) Altered surfactant production is found in sudden infant death syndrome.
(B) Paradoxical breathing occurs when a negative intrathoracic pressure is transmitted to the
abdomen by a weakened, poorly functioning diaphragm.
(C) Asthma is characterized by spasm and constriction of the airways resulting in increased
resistance to airflow.
(D) If the pulmonary tree is obstructed for any reason, inspired air has difficulty overcoming
the resistance and getting out. The rate of respiration increases in order to compensate, thus
increasing air exchange.
QUESTION 138
A 5-year-old child has suffered second-degree thermal burns over 30% of her body. Fortyeight hours after the burn injury, the nurse must begin to monitor the child for which one of
the following complications?
A. Fluid volume deficit
B. Fluid volume excess
C. Decreased cardiac output
D. Severe hypotension
Answer: B
Explanation:
(A) Fluid volume deficit resulting from fluid shifts to the interstitial spaces occurs in the first
48 hours.
(B) Forty-eight hours to 72 hours after the burn injury and fluid resuscitation, capillary
permeability is restored and fluid requirements decrease. Interstitial fluid returns rapidly to
the vascular compartment, and the nurse must monitor the child for signs and symptoms of
hypervolemia.
(C) Increased cardiac output results as fluids shift back to the vascular compartment.
(D) Hypertension is the result of hypervolemia.
QUESTION 139
Prenatal clients are routinely monitored for early signs of pregnancy-induced hypertension
(PIH). For the prenatal client, which of the following blood pressure changes from baseline
would be most significant for the nurse to report as indicative of PIH?
A. 136/88 to 144/93
B. 132/78 to 124/76

C. 114/70 to 140/88
D. 140/90 to 148/98
Answer: C
Explanation:
(A) These blood pressure changes reflect only an 8 mm Hg systolic and a 5 mm Hg diastolic
increase, which is insufficient for blood pressure changes indicating PIH.
(B) These blood pressure changes reflect a decrease in systolic pressure of 8 mm Hg and
diastolic pressure of 2 mm Hg; these values are not indicative of blood pressure increases
reflecting PIH.
(C) The definition of PIH is an increase in systolic blood pressure of 30 mm Hg and/or
diastolic blood pressure of 15 mm Hg. These blood pressures reflect a change of 26 mm Hg
systolically and 18mm Hg diastolically.
(D) These blood pressures reflect a change of only 8 mm Hg systolically and 8 mm Hg
diastolically, which is insufficient for blood pressure changes indicating PIH.
QUESTION 140
An 83-year-old client has been hospitalized following a fall in his home. He has developed a
possible fecal impaction. Which of the following assessment findings would be most
indicative of a fecal impaction?
A. Boardlike, rigid abdomen
B. Loss of the urge to defecate
C. Liquid stool
D. Abdominal pain
Answer: C
Explanation:
(A) A board like, rigid abdomen would point to a perforated bowel, not a fecal impaction.
(B) When a client is fecally impacted, a common symptom is the urge to defecate but the
inability to do so.
(C) When an impaction is present, only liquid stool will be able to pass around the impacted
site.
(D) Abdominal pain without distention is not a sign of a fecal impaction.

QUESTION 141
A postpartum client complains of rectal pressure and severe pain in her perineum; this may be
indicative of:
A. Afterbirth pains
B. Constipation
C. Cystitis
D. A hematoma of the vagina or vulva
Answer: D
Explanation:
(A) Afterbirth pains are a common complaint in the postpartum client, but they are located in
the uterus.
(B) Constipation may cause rectal pressure but is not usually associated with “severe pain”.
(C) Cystitis may cause pain, but the location is different.
(D) Hematomas are frequently associated with severe pain and pressure. Further assessments
are indicated for this client.
QUESTION 142
Iron dextran (Imferon) is a parenteral iron preparation.
The nurse should know that it:
A. Is also called intrinsic factor
B. Must be given in the abdomen
C. Requires use of the Z-track method
D. Should be given SC
Answer: C
Explanation:
(A) Intrinsic factor is needed to absorb vitamin B12.
(B) Iron dextran is given parenterally, but Z-track in a large muscle.
(C) A Ztrack method of injection is required to prevent staining and irritation of the tissue.
(D) An SC injection is not deep enough and may cause subcutaneous fat abscess formation.
QUESTION 143
The nurse would be sure to instruct a client on the signs and symptoms of an eye infection
and hemorrhage. These signs and symptoms would include:
A. Blurred vision and dizziness

B. Eye pain and itching
C. Feeling of eye pressure and headache
D. Eye discharge and hemoptysis
Answer: B
Explanation:
(A) Although blurred vision may occur, dizziness would not be associated with an infection
or hemorrhage.
(B) Eye pain is a symptom of hemorrhage within the eye, and itching is associated with
infection.
(C) Nausea and headache would not be usual symptoms of eye hemorrhage or infection.
(D) Some eye discharge might be anticipated if an infection is present; hemoptysis would not.
QUESTION 144
The nurse is caring for a laboring client. Assessment data include cervical dilation 9 cm;
contractions every 1–2 minutes; strong, large amount of “bloody show”. The most
appropriate nursing goal for this client would be:
A. Maintain client’s privacy.
B. Assist with assessment procedures.
C. Provide strategies to maintain client control.
D. Enlist additional caregiver support to ensure client’s safety.
Answer: C
Explanation:
(A) Privacy may help the laboring client feel safer, but measures that enhance coping take
priority.
(B) The frequency of assessments do increase in transition, but helping the client to maintain
control and cope with this phase of labor takes on importance.
(C) This laboring client is in transition, the most difficult part of the first stage of labor
because of decreased frequency, increased duration and intensity, and decreased resting phase
of the uterine contraction. The client’s ability to cope is most threatened during this phase of
labor, and nursing actions are directed toward helping the client to maintain control.
(D) Safety is a concern throughout labor, but helping the client to cope takes on importance in
transition.

QUESTION 145
A complication for which the nurse should be alert following a liver biopsy is:
A. Hepatic coma
B. Jaundice
C. Ascites
D. Shock
Answer: D
Explanation:
(A) Hepatic coma may occur in liver disease due to the increased NH3 levels, not due to liver
biopsy.
(B) Jaundice may occur due to increased bilirubin levels, not due to liver biopsy.
(C) Ascites would occur due to portal hypertension, not due to liver biopsy.
(D) Hemorrhage and shock are the most likely complications after liver biopsy because of
already existing bleeding tendencies in the vascular makeup of the liver.
QUESTION 146
A 42-year-old client on an inpatient psychiatric unit comments that he was brought to the
hospital by his wife because he had taken too many pills and states, “I just couldn’t take it
anymore.” The nurse’s best response to this disclosure would be:
A. “You shouldn’t do things like that, just tell someone you feel bad.”
B. “Tell me more about what you couldn’t take anymore.”
C. “I’m sure you probably didn’t mean to kill yourself.”
D. “How long have you been in the hospital.”
Answer: B
Explanation:
(A) Disapproving gives the impression that the nurse has a right to pass judgment on the
client’s thoughts, actions, or ideas.
(B) Giving a broad opening gives the client encouragement to continue with verbalization.
(C) Failing to acknowledge the client’s feelings conveys a lack of understanding and
empathy.
(D) Changing the subject takes the conversation away from the client and is indicative of the
nurse’s anxiety or insensitivity.

QUESTION 147
After a 10-year-old child with insulin-dependent diabetes mellitus receives her dinner tray,
she tells the nurse that she hates broccoli and wants some corn on the cob. The nurse’s
appropriate response is:
A. “No vegetable exchanges are allowed.”
B. “Corn and other starchy vegetables are considered to be bread exchanges.”
C. “Yes, you may exchange any vegetable for any other vegetable.”
D. “Yes, but only one-half ear is allowed.”
Answer: B
Explanation:
(A) Sites for injection need to be rotated, including abdominal sites, to enhance insulin
absorption.
(B) The pinch technique is the most effective method for obtaining skin tightness to allow
easy entrance of the needle to subcutaneous tissues.
(C) Massaging the site of injection facilitates absorption of the insulin.
(D) Changing the needle will break the sterility of the system. It has become acceptable
practice to reuse disposable needles and syringes for 3–7 days.
QUESTION 148
Which of the following signs might indicate a complication during the labor process with
vertex presentation?
A. Fetal tachycardia to 170 bpm during a contraction
B. Nausea and vomiting at 8–10 cm dilation
C. Contraction lasting 60 seconds
D. Appearance of dark-colored amniotic fluid
Answer: D
Explanation:
(A) Fetal tachycardia may indicate fetal hypoxia; however, 170 bpm is only mild tachycardia.
(B) Nausea and vomiting occur frequently during transition and are not a complication.
(C) Contractions frequently last 60–90 seconds during the transitional phase of labor and are
not considered a complication as long as the uterus relaxes completely between contractions.
(D) Passage of meconium in a vertex presentation is a sign of fetal distress; this may be
normal in a breech presentation owing to pressure on the presenting part.

QUESTION 149
The nurse would be concerned if a client exhibited which of the following symptoms during
her postpartum stay?
A. Pulse rate of 50–70 bpm by her third postpartum day
B. Diuresis by her second or third postpartum day
C. Vaginal discharge or rubra, serosa, then rubra
D. Diaphoresis by her third postpartum day
Answer: C
Explanation:
(A) Bradycardia is an expected assessment during the postpartum period.
(B) Diuresis can occur during labor and the postpartum period and is an expected
physiological adaptation.
(C) A return of rubra after the serosa period may indicate a postpartal complication.
(D) Diaphoresis, especially at night, is an expected physiological change and does not
indicate an infectious process. Bradycardia, diuresis, and diaphoresis are normal postpartum
physiological responses to adjust the cardiac output and blood volume to the nonpregnant
state.
QUESTION 150
MgSO4 blood levels are monitored and the nurse would be prepared to administer the
following antidote for MgSO4 side effects or toxicity:
A. Magnesium oxide
B. Calcium hydroxide
C. Calcium gluconate
D. Naloxone (Narcan)
Answer: C
Explanation:
(A, B) These drugs are not antidotes for MgSO4.
(C) This drug is the standard antidote and should always be readily available when MgSO4 is
being administered.
(D) This drug is an antidote for narcotics, not MgSO4.

QUESTION 151
A mother brings her 3-year-old child who is unconscious but breathing to the ER with an
apparent drug overdose. The mother found an empty bottle of aspirin next to her child in the
bathroom. Which nursing action is the most appropriate?
A. Put in a nasogastric tube and lavage the child’s stomach.
B. Monitor muscular status.
C. Teach mother poison prevention techniques.
D. Place child on respiratory assistance.
Answer: A
Explanation:
(A) The immediate treatment for drug overdose is removal of the drug from the stomach by
either forced emesis or gastric lavage. The child’s unconscious state prohibits forced emesis.
(B) Toxic amounts of salicylates directly affect the respiratory system, which could lead to
respiratory failure.
(C) The mother’s anxiety is probably so high that preventive guidance will be ineffective.
(D) Respiratory assistance is not needed if the child’s respiratory function is unaltered.
QUESTION 152
Azulfidine (Sulfasalazine) may be ordered for a client who has ulcerative colitis. Which of
the following is a nursing implication for this drug?
A. Limit fluids to 500 mL/day.
B. Administer 2 hours before meals.
C. Observe for skin rash and diarrhea.
D. Monitor blood pressure, pulse.
Answer: C
Explanation:
(A) Fluids up to 2500–3000 mL/day are needed to prevent kidney stones.
(B) The client should be instructed to take oral preparations with meals or snacks to lessen
gastric irritation.
(C) Sulfasalazine causes skin rash and diarrhea.
(D) Blood pressure and pulse are not altered by sulfasalazine.
QUESTION 153
A dose of theophylline may need to be altered if a client with COPD:

A. Is allergic to morphine
B. Has a history of arthritis
C. Operates machinery
D. Is concurrently on cimetidine for ulcers
Answer: D
Explanation:
(A) The effects of morphine or an allergic response to the drug will not affect theophylline
clearance.
(B) Xanthines are used cautiously in clients with severe cardiac disease, liver disease, cor
pulmonale, hypertension, or hyperthyroidism. Arthritis does not influence the dosage of
theophylline.
(C) Theophylline does not cause sedation or drowsiness.
Conversely, its side effects may be exhibited by central nervous system stimulation.
(D) Cimetidine decreases theophylline clearance from the system and increases theophylline
levels in the blood, thus increasing the risk of toxicity.
QUESTION 154
A 56-year-old client is admitted to the psychiatric unit in a state of total despair. She feels
hopeless and worthless, has a flat affect and very sad appearance, and is unable to feel
pleasure from anything. Her husband has been assisting her at home with the housework and
cooking; however, she has not been eating much, lies around or sits in a chair most of the day,
and is becoming confused and thinks her family does not want her around anymore. In
assessing the client, the nurse determines that her behavior is consistent with:
A. Transient depression
B. Mild depression
C. Moderate depression
D. Severe depression
Answer: D
Explanation:
(A) Transient depression manifests as sadness or the “blues” as seen with everyday
disappointments and is not necessarily dysfunctional.
(B) Mild depression manifests as symptoms seen with grief response, such as denial, sadness,
withdrawal, somatic symptoms, and frequent or continuous thoughts of the loss.

(C) Moderate depression manifests as feelings of sadness, negativism; low self-esteem;
rumination about life’s failures; decreased interest in grooming and eating; and possibly sleep
disturbances. These symptoms are consistent with dysthymia.
(D) Severe depression manifests as feelings of total despair, hopelessness, emptiness,
inability to feel pleasure; possibly extreme psychomotor retardation; inattention to hygiene;
delusional thinking; confusion; self- blame; and suicidal thoughts. These symptoms are
consistent with major depression.
QUESTION 155
A client is pregnant with her second child. Her last menstrual period began on January 15.
Her expected date of delivery would be:
A. October 8
B. October 15
C. October 22
D. October 29
Answer: C
Explanation:
(A) Incorrect application of Nägele’s rule: correctly subtracted 3 months but subtracted 7
days rather than added.
(B) Incorrect application of Nägele’s rule: correctly subtracted 3 months but did not add 7
days.
(C) Correct application of Nägele’s rule: correctly subtracted 3 months and added 7 days.
(D) Incorrect application of Nägele’s rule: correctly subtracted 3 months but added 14 days
instead of 7 days.
QUESTION 156
Which of the following changes in blood pressure readings should be of greatest concern to
the nurse when assessing a prenatal client?
A. 130/88 to 144/92
B. 136/90 to 148/100
C. 150/96 to 160/104
D. 118/70 to 130/88
Answer: D
Explanation:

(A, B, C) The individual’s systolic and diastolic changes are more significant than the
relatively high initial blood pressure readings.
(D) The systolic pressure went up 12 mm Hg and the diastolic pressure 18 mm Hg. This is a
more significant rise than the increases in A–C choices, and client should receive more
frequent evaluations and care.
QUESTION 157
The nurse would assess the client’s correct understanding of the fertility awareness methods
that enhance conception, if the client stated that:
A. “My sexual partner and I should have intercourse when my cervical mucosa is thick and
cloudy.”
B. “At ovulation, my basal body temperature should rise about 0.5°F.”
C. “I should douche immediately after intercourse.”
D. “My sexual partner and I should have sexual intercourse on day 14 of my cycle regardless
of the length of the cycle.”
Answer: B
Explanation:
(A) At ovulation, the cervical mucus is increased, stretchable, and watery clear.
(B) Under the influence of progesterone, the basal body temperature increases slightly after
ovulation.
(C) To enhance fertility, measures should be taken that promote retention of sperm rather than
removal.
(D) Ovulation, the optimal time for conception, occurs 14 + 2 days before the next menses;
therefore, the date of ovulation is directly related to the length of the menstrual cycle.
QUESTION 158
In evaluating the laboratory results of a client with severe pressure ulcers, the nurse finds that
her albumin level is low. A decrease in serum albumin would contribute to the formation of
pressure ulcers because:
A. The proteins needed for tissue repair are diminished.
B. The iron stores needed for tissue repair are inadequate.
C. A decreased serum albumin level indicates kidney disease.
D. A decreased serum albumin causes fluid movement into the blood vessels, causing
dehydration.

Answer: A
Explanation:
(A) Serum albumin levels indicate the adequacy of protein stores available for tissue repair.
(B) Serum albumin does not measure iron stores.
(C) Serum albumin levels do not measure kidney function.
(D) A decreased serum albumin level would cause fluid movement out of blood vessels, not
into them.
QUESTION 159
A client is being discharged on warfarin (Coumadin), an oral anticoagulant. The nurse
instructs him about using this drug. Which following response by the client indicates the need
for further teaching?
A. “I should shave with my electric razor while on Coumadin.”
B. “I will inform my dentist that I am on anticoagulant therapy before receiving dental work.”
C. “I will continue with my usual dosage of aspirin for my arthritis when I return home.”
D. “I will wear an ID bracelet stating that I am on anticoagulants.”
Answer: C
Explanation:
(A) Using an electric razor prevents the risk of cuts while shaving.
(B) Any physician or dentist should be informed of anticoagulant therapy because of the risk
of bleeding due to a prolonged PT.
(C) The client should be instructed to consult with his physician. Aspirin is avoided because it
potentiates the effects of oral anticoagulants by interfering with platelet aggregation.
(D) Identification bracelets are necessary to direct treatment, especially in an emergency
situation.
QUESTION 160
When a client with pancreatitis is discharged, the nurse needs to teach him how to prevent
another occurrence of acute pancreatitis. Which of the following statements would indicate he
has an understanding of his disease?
A. “I will not eat any raw or uncooked vegetables.”
B. “I will limit my alcohol to one cocktail per day.”
C. “I will look into attending Alcoholics Anonymous meetings.”
D. “I will report any changes in bowel movements to my doctor.”

Answer: C
Explanation:
(A) Raw or uncooked vegetables are all right to eat post discharge.
(B) This client must avoid any alcohol intake.
(C) The client displays awareness of the need to avoid alcohol.
(D) This action would be pertinent only if fatty stools associated with chronic hepatitis were
the problem.
QUESTION 161
A client decided early in her pregnancy to breast-feed her first baby. She gave birth to a
normal, full-term girl and is now progressing toward the establishment of successful lactation.
To remove the baby from her breast, she should be instructed to:
A. Gently pull the infant away
B. Withdraw the breast from the infant’s mouth
C. Compress the areolar tissue until the infant drops the nipple from her mouth
D. Insert a clean finger into the baby’s mouth beside the nipple
Answer: D
Explanation:
(A) In pulling the infant away from the breast without breaking suction, nipple trauma is
likely to occur.
(B) In pulling the breast away from the infant without breaking suction, nipple trauma is
likely to occur.
(C) Compressing the maternal tissue does not break the suction of the infant on the breast and
can cause nipple trauma.
(D) By inserting a finger into the infant’s mouth beside the nipple, the lactating mother can
break the suction and the nipple can be removed without trauma.
QUESTION 162
A parent told the public health nurse that her 6-year-old son has been taking tetracycline for a
chronic skin condition. The parent asked if this could cause any problems for the child. What
should the nurse explain to the parent?
A. Giving tetracycline to a child younger than 8 years may cause permanent staining of his
teeth.
B. If you give tetracycline with milk, it may be absorbed readily.

C. The medication should be given to adults, not children.
D. Secondary infections of chronic skin disorders do not respond to antibiotics.
Answer: A
Explanation:
(A) Tetracycline should be avoided during tooth development because it interferes with
enamel formation and dental pigmentation.
(B) Milk interferes with the absorption of tetracyclines.
(C) Children older than 9 years or past the tooth development stage may be given
tetracycline.
(D) Secondary infections of chronic skin disorders may respond to antibiotics such as
penicillin or tetracyclines.
QUESTION 163
MgSO4 is ordered IV following the established protocol for a client with severe PIH. The
anticipated effects of this therapy are anticonvulsant and:
A. Vasoconstrictive
B. Vasodilative
C. Hypertensive
D. Antiemetic
Answer: B
Explanation:
(A) An anticonvulsant effect is the goal of drug therapy for PIH. However, we would not
want to increase the vasoconstriction that is already present. This would make the symptoms
more severe.
(B) An anticon-vulsant effect and vasodilation are the desired outcomes when administering
this drug.
(C) An anticonvulsant effect is the goal of drug therapy for PIH; however, hypertensive drugs
would increase the blood pressure even more.
(D) An anticonvulsant effect is the goal of drug therapy for PIH. MgSO4 is not classified as
an antiemetic. Antiemetics are not indicated for PIH treatment.

QUESTION 164
The nurse is preparing a 6-year-old child for an IV insertion. Which one of the following
statements by the nurse is appropriate when preparing a child for a potentially painful
procedure?
A. “Some say this feels like a pinch or a bug bite. You tell me what it feels like.”
B. “This is going to hurt a lot; close your eyes and hold my hand.”
C. “This is a terrible procedure, so don’t look.”
D. “This will hurt only a little; try to be a big boy.”
Answer: A
Explanation:
(A) Educating the child about the pain may lessen anxiety. The child should be prepared for a
potentially painful procedure but avoid suggesting pain. The nurse should allow the child his
own sensory perception and evaluation of the procedure.
(B) The nurse should avoid absolute descriptive statements and allow the child his own
perception of the procedure.
(C) The nurse should avoid evaluative statements or descriptions and give the child control in
describing his reactions.
(D) False statements regarding a painful procedure will cause a loss of trust between the child
and the nurse.
QUESTION 165
A 74-year-old female client is 3 days postoperative. She has an indwelling catheter and has
been progressing well. While the nurse is in the room, the client states, “Oh dear”, I feel like I
have to urinate again! Which of the following is the most appropriate initial nursing
response?
A. Assure her that this is most likely the result of bladder spasms.
B. Check the collection bag and tubing to verify that the catheter is draining properly.
C. Instruct her to do Kegel exercises to diminish the urge to void.
D. Ask her if she has felt this way before.
Answer: B
Explanation:
(A) Although this may be an appropriate response, the initial response would be to assure the
patency of the catheter.

(B) The most frequent reason for an urge to void with an indwelling catheter is blocked
tubing. This response would be the best initial response.
(C) Kegel exercises while a retention catheter is in place would not help to prevent a voiding
urge and could irritate the urethral sphincter.
(D) Though the nurse would want to ascertain whether the client has felt the same urge to
void before, the initial response should be to assure the patency of the catheter.
QUESTION 166
A six-month-old infant is receiving ribavirin for the treatment of respiratory syncytial virus.
Ribavirin is administered via which one of the following routes?
A. Oral
B. IM
C. IV
D. Aerosol
Answer: D
Explanation:
(A) Ribavirin is not supplied in an oral form.
(B) Ribavirin is administered by aerosol in order to decrease the duration of viral shedding
within the infected tissue.
(C) Ribavirin is not approved for IV use to treat respiratory syncytial virus.
(D) Ribavirin is a synthetic antiviral agent supplied as a crystalline powder that is
reconstituted with sterile water. A Small Aerosol Particle Generator unit aerosolizes the
medication for delivery by oxygen hood, croup tent, or aerosol mask.
QUESTION 167
A pregnant client is at the clinic for a third trimester prenatal visit. During this examination, it
has been determined that her fetus is in a vertex presentation with the occiput located in her
right anterior quadrant. On her chart this would be noted as:
A. Right occipitoposterior
B. Right occipitoanterior
C. Right Sacro anterior
D. LOA
Answer: B
Explanation:

(A) The fetus in the right occipitoposterior position would be presenting with the occiput in
the maternal right posterior quadrant.
(B) Fetal position is defined by the location of the fetal presenting part in the four quadrants
of the maternal pelvis. The right occipitoanterior is a fetus presenting with the occiput in
mother’s right anterior quadrant.
(C) The fetus in right Sacro anterior position would be presenting a sacrum, not an occiput.
(D) The fetus in left occipitoanterior position would be presenting with the occiput in the
mother’s left anterior quadrant.
QUESTION 168
Which of the following menu choices would indicate that a client with pressure ulcers
understands the role diet plays in restoring her albumin levels?
A. Broiled fish with rice
B. Bran flakes with fresh peaches
C. Lasagna with garlic bread
D. Cauliflower and lettuce salad
Answer: A
Explanation:
(A) Broiled fish and rice are both excellent sources of protein.
(B) Fresh fruits are not a good source of protein.
(C) Foods in the bread group are not high in protein.
(D) Most vegetables are not high in protein; peas and beans are the major vegetables higher in
protein.
QUESTION 169
A 42-year-old client with bipolar disorder has been hospitalized on the inpatient psychiatric
unit. She is dancing around, talking incessantly, and singing. Much of the time the client is
anorexic and eats very little from her tray before she is up and about again. The nurse’s
intervention would be to:
A. Confront the client with the fact that she will have to eat more from her tray to sustain her
B. Try to get the client to focus on her eating by offering a detailed discussion on the
importance of nutrition
C. Let her have snacks and drinks anytime that she wants them because she will not eat at
regular meal times

D. Not expect the client to sit down for complete meals; monitor intake, offering snacks and
juice frequently
Answer: D
Explanation:
(A) The manic client’s mood may easily change from euphoric to irritable. The nurse should
avoid confrontation and let the client know what she can do, rather than what she cannot.
(B) Although helpful to refocus or redirect the manic client to discuss only one topic at a
time, distractibility is very high and it’s best to avoid long discussions.
(C) Manic clients have a tendency to manipulate persons in their environment. Staff should
monitor intake, including at mealtime and snacks, and be consistent in their approach to
meeting nutritional needs.
(D) Manic clients may not sit and eat complete meals, but they can carry foods and liquids
from regular meals with them. Staff can monitor and give high-caloric and high-energy
snacks and liquids.
QUESTION 170
A 35-weeks-pregnant client is undergoing a nonstress test (NST). During the 20-minute
examination, the nurse notes three fetal movements accompanied by accelerations of the fetal
heart rate, each 15 bpm, lasting 15 seconds. The nurse interprets this test to be:
A. Nonreactive
B. Reactive
C. Positive
D. Negative
Answer: B
Explanation:
(A) In a nonreactive NST, the criteria for reactivity are not met.
(B) A reactive NST shows at least two accelerations of FHR with fetal movements, each 15
bpm, lasting 15 seconds or more, over 20 minutes.
(C, D) This term is used to interpret a contraction stress test (CST), or oxytocin challenge test,
not an NST.
QUESTION 171
A 30-year-old client has just been treated in the ER for bruises and abrasions to her face and a
broken arm from domestic violence, which has been increasing in frequency and intensity

over the last few months. The nurse assesses her as being very anxious, fearful, bewildered,
and feeling helpless as she states, “I don’t know what to do”, I’m afraid to go home. The best
response by the nurse to the client would be:
A. “I wouldn’t want to go home either; call a friend who could help you.”
B. “Did you do something that could have made him so angry?”
C. “Let’s talk about people and resources available to you so that you don’t have to go home.”
D. “I’ll call the police and they will take care of him, and you can go home and get some
rest.”
Answer: C
Explanation:
(A) A person in crisis needs support, assistance, and direction from a caregiver rather than
just an instruction.
(B) A battered person may feel guilt and think that they cause the abuser’s behavior; however,
the abuser has the problem and goes through phases of violence.
(C) The nurse should provide support and guidance to the client in crisis by offering
alternatives and assist in referrals.
(D) Focusing on help from law enforcement may be a very temporary solution, because the
victim may be fearful of pressing charges. This answer does not address the crisis of going
home.
QUESTION 172
A 74-year-old obese man who has undergone open reduction and internal fixation of the right
hip is 8 days postoperative. He has a history of arthritis and atrial fibrillation. He admits to
right lower leg pain, described as “a cramp in my leg”. An appropriate nursing action is to:
A. Assess for pain with plantarflexion
B. Assess for edema and heat of the right leg
C. Instruct him to rub the cramp out of his leg
D. Elevate right lower extremity with pillows propped under the knee
Answer: B
Explanation:
(A) Calf pain with dorsiflexion of the foot (Homan’s sign) can be a sign of a deep venous
thrombosis; however, it is not diagnostic of the condition.
(B) Swelling and warmth along the affected vein are commonly observed clinical
manifestations of a deep venous thrombosis as a result of inflammation of the vessel wall.

(C) Rubbing or massaging of the affected leg is contraindicated because of the risk of the clot
breaking loose and becoming an embolus.
(D) A pillow behind the knee can be constricting and further impair blood flow.
QUESTION 173
A 26-year-old client is admitted to the labor, delivery, recovery, postpartum unit. The nurse
completes her assessment and determines the client is in the first stage of labor. The nurse
should instruct her:
A. To hold her breath during contractions
B. To be flat on her back
C. Not to push with her contractions
D. To push before becoming fully dilated
Answer: C
Explanation:
(A) This nursing action may cause hyperventilation.
(B) This nursing action could cause inferior vena cava syndrome.
(C) The client is allowed to push only after complete dilation during the second stage of labor.
The nurse needs to know the stages of labor.
(D) If the client pushes before dilation, it could cause cervical edema and/or edema to the
fetal scalp; both of these could contribute to increased risk of complications.
QUESTION 174
A client in active labor asks the nurse for coaching with her breathing during contractions.
The client has attended Lamaze birth preparation classes. Which of the following is the best
response by the nurse?
A. “Keep breathing with your abdominal muscles as long as you can.”
B. “Make sure you take a deep cleansing breath as the contractions start, focus on an object,
and breathe about 16–20 times a minute with shallow chest breaths.”
C. “Find a comfortable position before you start a contraction. Once the contraction has
started, take slow breaths using your abdominal muscles.”
D. “If a woman in labor listens to her body and takes rapid, deep breaths, she will be able to
deal with her contractions quite well.”
Answer: B
Explanation:

(A) Lamaze childbirth preparation teaches the use of chest, not abdominal, breathing.
(B) In Lamaze preparation, every patterned breath is preceded by a cleansing breath; as labor
progresses, shallow, paced breathing is found to be effective.
(C) It is important to assume a comfortable position in labor, but the Lamaze prepared
laboring woman is taught to breathe with her chest, not abdominal, muscles.
(D) When deep chest breathing patterns are used in Lamaze preparation, they are slowly
paced at a rate of 6–9 breaths/min.
QUESTION 175
A couple is planning the conception of their first child.
The wife, whose normal menstrual cycle is 34 days in length, correctly identifies the time that
she is most likely to ovulate if she states that ovulation should occur on day:
A. 14 + 2 days
B. 16 + 2 days
C. 20 + 2 days
D. 22 + 2 days
Answer: C
Explanation:
(A) Ovulation is dependent on average length of menstrual cycle, not standard 14 days.
(B) Ovulation occurs 14 + 2 days before next menses (34 minus 14 does not equal 16).
(C) Ovulation occurs 14 + 2 days before next menses (34 minus 14 equals 20).
(D) Ovulation occurs 14 + 2 days before next menses (34 minus 14 does not equal 22).
QUESTION 176
A nurse should carefully monitor a client for the following side effect of MgSO4:
A. Visual blurring
B. Tachypnea
C. Epigastric pain
D. Respiratory depression
Answer: D
Explanation:
(A, C) The nurse should provide good distractors because these symptoms indicate that PIH
has become more severe and may precede the convulsive or eclamptic phase.
(B) This is the opposite side effect of this medication.

(D) This is a common side effect of this medication and needs to be monitored and recorded
frequently.
QUESTION 177
The nurse would teach a male client ways to minimize the risk of infection after eye surgery.
Which of the following indicates the client needs further teaching?
A. “I will wash my hands before instilling eye medications.”
B. “I will wear sunglasses when going outside.”
C. “I will wear an eye patch for the first 3 postoperative days.”
D. “I will maintain the sterility of the eye medications.”
Answer: C
Explanation:
(A) Hand washing would be an important action designed to prevent transmission of
pathogens from the hands to the eye.
(B) Wearing sunglasses when going outside will prevent airborne pathogens from entering the
eye.
(C) Eye patches are most frequently ordered to be worn while the client sleeps or naps, not
constantly for this length of time.
(D) Eye medications are sterile; clients need to be taught how to maintain this sterility.
QUESTION 178
With a geriatric client, the nurse should also assess whether he has been obtaining a yearly
vaccination against influenza. Why is this assessment important?
A. Influenza is growing in our society.
B. Older clients generally are sicker than others when stricken with flu.
C. Older clients have less effective immune systems.
D. Older clients have more exposure to the causative agents.
Answer: C
Explanation:
(A) Although influenza is common, the elderly are more at risk because of decreased
effectiveness of their immune system, not because the incidence is increasing.
(B) Older clients have the same degree of illness when stricken as other populations.
(C) As people age, their immune system becomes less effective, increasing their risk for
influenza.

(D) Older clients have no more exposure to the causative agents than do school-age children,
for example.
QUESTION 179
A client delivered her first-born son 4 hours ago. She asks the nurse what the white cheese
like substance is under the baby’s arms. The nurse should respond:
A. “This is a normal skin variation in newborns. It will go away in a few days.”
B. “Let me have a closer look at it. The baby may have an infection.”
C. “This material, called vernix, covered the baby before it was born. It will disappear in a
few days.”
D. “Babies sometimes have sebaceous glands that get plugged at birth. This substance is an
example of that condition.”
Answer: C
Explanation:
(A) This response identifies the fact that vernix is a normal neonatal variation, but it does not
teach the client medical terms that may be useful in understanding other healthcare personnel.
(B) This response may raise maternal anxiety and incorrectly identifies a normal neonatal
variation.
(C) This response correctly identifies this neonatal variation and helps the client to understand
medical terms as well as the characteristics of her newborn.
(D) Blocked sebaceous glands produce milia, particularly present on the nose.
QUESTION 180
A 26-year-old client is in a treatment center for aprazolam (Xanax) abuse and continues to
manifest moderate levels of anxiety 3 weeks into the rehabilitation program, often requesting
medication for “his nerves”. Included in the client’s plan of care is to identify alternate
methods of coping with stress and anxiety other than use of medication. After intervening
with assistance in stress reduction techniques, identifying feelings and past coping, the nurse
evaluates the outcome as being met if:
A. Client promises that he will not abuse aprazolam after discharge
B. Client demonstrates use of exercise or physical activity to handle nervous energy
following conflicts of everyday life
C. Client is able to verbalize effects of substance abuse on the body
D. Client has remained substance free during hospitalization and is discharged

Answer: B
Explanation:
(A) This client response does not address stress reduction techniques. Verbal response focuses
only on the problem.
(B) Exercise or physical activity is a common strategy or coping technique used to reduce
stress and anxiety.
(C) Verbalizing effects of substance abuse on the body may help with insight and break
through denial, but it is not a strategy to reduce anxiety.
(D) Remaining substance-free does indicate motivation to change lifestyle of substance abuse
or dependence, and it is not a stress reduction strategy in itself.
QUESTION 181
In assessing the nature of the stool of a client who has cystic fibrosis, what would the nurse
expect to see?
A. Clay-colored stools
B. Steatorrhea stools
C. Dark brown stools
D. Blood-tinged stools
Answer: B
Explanation:
(A) Clay-colored stools indicate dysfunction of the liver or biliary tract.
(B) In the early stages of cystic fibrosis, fat absorption is primarily affected resulting in fat,
foul, frothy, bulky stools.
(C) Dark brown stools indicate normal passage through the colon.
(D) Blood-tinged stools indicate dysfunction of the gastrointestinal (GI) tract.
QUESTION 182
A client is taught to eat foods high in potassium. Which food choices would indicate that this
teaching has been successful?
A. Pork chop, baked acorn squash, brussel sprouts
B. Chicken breast, rice, and green beans
C. Roast beef, baked potato, and diced carrots
D. Tuna casserole, noodles, and spinach
Answer: A

Explanation:
(A) Both acorn squash and brussels sprouts are potassium-rich foods.
(B) None of these foods is considered potassium rich.
(C) Only the baked potato is a potassium-rich food.
(D) Spinach is the only potassium-rich food in this option.
QUESTION 183
The nurse is teaching a 10-year-old insulin-dependent diabetic how to administer insulin.
Which one of the following steps must be taught for insulin administration?
A. Never use abdominal site for a rotation site.
B. Pinch the skin up to form a subcutaneous pocket.
C. Avoid applying pressure after injection.
D. Change needles after injection.
Answer: B
Explanation:
(A) Sites for injection need to be rotated, including abdominal sites, to enhance insulin
absorption.
(B) The pinch technique is the most effective method for obtaining skin tightness to allow
easy entrance of the needle to subcutaneous tissues.
(C) Massaging the site of injection facilitates absorption of the insulin.
(D) Changing the needle will break the sterility of the system. It has become acceptable
practice to reuse disposable needles and syringes for 3–7 days.
QUESTION 184
A client is experiencing muscle weakness and lethargy. His serum K+ is 3.2. What other
symptoms might he exhibit?
A. Tetany
B. Dysrhythmias
C. Numbness of extremities
D. Headache
Answer: B
Explanation:
(A) Tetany is seen with low calcium.

(B) Low potassium causes dysrhythmias because potassium is responsible for cardiac muscle
activity.
(C) Numbness of extremities is seen with high potassium.
(D) Headache is not associated with potassium excess or deficiency.
QUESTION 185
Other drugs may be ordered to manage a client’s ulcerative colitis. Which of the following
medications, if ordered, would the nurse question?
A. Methylprednisolone sodium succinate (Solu-Medrol)
B. Loperamide (Imodium)
C. Psyllium
D. 6-Mercaptopurine
Answer: D
Explanation:
(A) Methylprednisolone sodium succinate is used for its anti-inflammatory effects.
(B) Loperamide would be used to control diarrhea.
(C) Psyllium may improve consistency of stools by providing bulk.
(D) An immunosuppressant such as 6- mercaptopurine is used for chronic unrelenting
Crohn’s disease.
QUESTION 186
An 8-week-old infant has been diagnosed with gastroesophageal reflux. The nurse is teaching
the infant’s mother to care for the infant at home. Which one of the following statements by
the nurse is appropriate regarding the infant’s home care?
A. “Lay the infant flat on her left side after feeding.”
B. “Feed the infant every 4 hours with half-strength formula.”
C. “Antacids need to be given an hour before feeding.”
D. “Play activities should be carried out before instead of after feedings.”
Answer: D
Explanation:
(A) Elevating the child’s head to a 30-degree angle is the recommended position for
gastroesophageal reflux. The supine position predisposes the child to aspiration.
(B) Small, frequent feedings with thickened formula are recommended to minimize vomiting.

(C) Antacids should be given at the same time as the feeding to improve their buffering
action.
(D) The infant should be kept still after feedings to reduce the risk of vomiting and aspiration.
Vigorous activities should be carried out before feedings.
QUESTION 187
A mother is unsure about the type of toys for her 17-month-old child. Based on knowledge of
growth and development, what toy would the nurse suggest?
A. A pull toy to encourage locomotion
B. A mobile to improve hand-eye coordination
C. A large toy with movable parts to improve pincer grasp
D. Various large colored blocks to teach visual discrimination
Answer: A
Explanation:
(A) Increased locomotive skills make push-pull toys appropriate for the energetic toddler.
(B) Infants progress from reflex activity through simple repetitive behaviors to imitative
behavior. Hand-eye coordination forms the foundation of other movements.
(C) At age 8 months, infants begin to have pincer grasp. Toys that help infants develop the
pincer grasp are recommended for this age group.
(D) Various large colored blocks are suggested toys for infants 6–12 months of age to help
visual stimulation.
QUESTION 188
A group of nursing students at a local preschool day care center are going to screen each
child’s fine and gross motor, language, and social skills. The students will use which one of
the most widely used screening tests?
A. Revised Pre-screening Developmental Questionnaire
B. Goodenough Draw-a-Person Screening Test
C. Denver Development Screening Test
D. Caldwell Home Inventory
Answer: C
Explanation:

(A) The Revised Pre-screening Developmental Questionnaire is more age appropriate and
offers simplified parent scoring and easier comparison. It is used by parents instead of
professionals.
(B) The Goodenough Draw- a-Person test is used to assess intellectual development.
(C) The Denver Developmental Screening Test is one of the most widely used screening tests.
It offers a concise, easy-to-administer, systematic approach to assessing the preschool child. It
is widely used because of its reliability and validity.
(D) The Caldwell Home Inventory is used to assess the home environment in areas of social,
emotional, and cognitive supports.
QUESTION 189
A gravida 2 para 1 client is hospitalized with severe preeclampsia. While she receives
magnesium sulfate (MgSO4) therapy, the nurse knows it is safe to repeat the dosage if:
A. Deep tendon reflexes are absent
B. Urine output is 20 mL/hr
C. MgSO4 serum levels are 15 mg/dL
D. Respirations are 16 breaths/min
Answer: D
Explanation:
(A) MgSO4 is a central nervous system depressant. Loss of reflexes is often the first sign of
developing toxicity.
(B) Urinary output at 25 mL/hr or 100 mL in 4 hours may result in the accumulation of toxic
levels of magnesium.
(C) The therapeutic serum range for MgSO4 is 6–8 mg/dL. Higher levels indicate toxicity.
(D) Respirations of 16 breaths/min indicate that toxic levels of magnesium have not been
reached. Medication administration would be safe.
QUESTION 190
In addition to changing the mother’s position to relieve cord pressure, the nurse may employ
the following measure (s) in the event that she observes the cord out of the vagina:
A. Immediately pour sterile saline on the cord, and repeat this every 15 minutes to prevent
drying.
B. Cover the cord with a wet sponge.
C. Apply a cord clamp to the exposed cord, and cover with a sterile towel.

D. Keep the cord warm and moist by continuous applications of warm, sterile saline
compresses.
Answer: D
Explanation:
(A) Saline should be warmed; waiting 15 minutes may not keep the cord moist.
(B) This choice does not specify what the sponge was “wet with”.
(C) This measure would stop circulation to the fetus.
(D) The cord should be kept warm and moist to maintain fetal circulation. This measure is an
accepted nursing action.
QUESTION 191
Following a gastric resection, which of the following actions would the nurse reinforce with
the client in order to alleviate the distress from dumping syndrome?
A. Eating three large meals a day
B. Drinking small amounts of liquids with meals
C. Taking a long walk after meals
D. Eating a low-carbohydrate diet
Answer: D
Explanation:
(A) Six small meals are recommended.
(B) Liquids after meals increase the time food empties from the stomach.
(C) Lying down after meals is recommended to prevent gravity from producing dumping.
(D) A low- carbohydrate diet will prevent a hypertonic bolus, which causes dumping.
QUESTION 192
A 54-year-old client is admitted to the hospital with a possible gastric ulcer. He is a heavy
smoker. When discussing his smoking habits with him, the nurse should advise him to:
A. Smoke low-tar, filtered cigarettes
B. Smoke cigars instead
C. Smoke only right after meals
D. Chew gum instead
Answer: C
Explanation:
(A, B, D) Cigarettes, cigars, and chewing gum would stimulate gastric acid secretion.

(C) Smoking on a full stomach minimizes effect of nicotine on gastric acid.
QUESTION 193
A client with severe PIH receiving MgSO4 is placed in a quiet, darkened room. The nurse
bases this action on the following understanding:
A. The client is restless.
B. The elevated blood pressure causes photophobia.
C. Noise or bright lights may precipitate a convulsion.
D. External stimuli are annoying to the client with PIH.
Answer: C
Explanation:
(A) The client may be anxious and hyperresponsive to stimuli but not necessarily restless.
(B) This is not a physiological response to an elevated blood pressure in PIH.
(C) The nurse must know the nursing measures that decrease the potential for convulsions. A
quiet, darkened room decreases stimuli and promotes rest.
(D) External stimuli might induce a convulsion but are not annoying to the client with PIH.
QUESTION 194
Assessment of parturient reveals the following: cervical dilation 6 cm and station 22; no
progress in the last 4 hours. Uterine contractions decreasing in frequency and intensity.
Marked molding of the presenting fetal head is described. The physician orders, “Begin
oxytocin” induction at 1 mU/min. The nurse should:
A. Begin the oxytocin induction as ordered
B. Increase the dosage by 2 mU/min increments at 15-minute intervals
C. Maintain the dosage when duration of contractions is 40–60 seconds and frequency is at
212–4 minute intervals
D. Question the order
Answer: D
Explanation:
(A) Oxytocin stimulates labor but should not be used until CPD (cephalopelvic disproportion)
is ruled out in a dysfunctional labor.
(B) This answer is the correct protocol for oxytocin administration, but the medication should
not be used until CPD is ruled out.

(C) This answer is the correct manner to interpret effective stimulation, but oxytocin should
not be used until CPD is ruled out.
(D) This answer is the appropriate nursing action because the scenario presents a
dysfunctional labor pattern that may be caused by CPD. Oxytocin administration is
contraindicated in CPD.
QUESTION 195
A pregnant client comes to the office for her first prenatal examination at 10 weeks. She has
been pregnant twice before; the first delivery produced a viable baby girl at 39 weeks 3 years
ago; the second pregnancy produced a viable baby boy at 36 weeks 2 years ago. Both
children are living and well. Using the gravida and para system to record the client’s
obstetrical history, the nurse should record:
A. Gravida 3 para 1
B. Gravida 3 para 2
C. Gravida 2 para 1
D. Gravida 2 para 2
Answer: B
Explanation:
(A) This answer is an incorrect application of gravida and para. The client has had two prior
deliveries of more than 20 week’s gestation; therefore, para equals 2, not 1.
(B) This answer is the correct application of gravida and para. The client is currently pregnant
for the third time (G = 3), regardless of the length of the pregnancy, and has had two prior
pregnancies with birth after the 20th week (P = 2), whether infant was alive or dead.
(C) This answer is an incorrect application of gravida and para. The client is currently
pregnant for the third time (G = 3, not 2); prior pregnancies lasted longer than 20 weeks
(therefore, P = 2, not 1).
(D) This is an incorrect application of gravida and para. Client is currently pregnant for third
time (G = 3, not 2).
QUESTION 196
Which of the following would indicate the need for further teaching for the client with
COPD? The client verbalizes the need to:
A. Eat high-calorie, high-protein foods
B. Take vitamin supplementation

C. Eliminate intake of milk and milk products
D. Eat small, frequent meals
Answer: C
Explanation:
(A) Protein is vital for the maintenance of muscle to aid in breathing. A high-calorie diet
using higher fat than carbohydrate content is given because clients are unable to breathe off
the excess CO2 that is an end product of carbohydrate metabolism.
(B) Inadequate nutritional status, in particular, deficiencies in vitamins A and C, decreases
resistance to infection.
(C) Milk does not make mucus thicker. It may coat the back of the throat and make it feel
thicker. Rinsing the mouth with water after drinking milk will prevent this problem.
(D) Small, frequent meals minimize a fullness sensation and reduce pressure on the
diaphragm. The work of breathing and SOB are also reduced.
QUESTION 197
The nurse provides a male client with diet teaching so that he can help prevent constipation in
the future. Which food choices indicate that this teaching has been understood?
A. Omelette and hash browns
B. Pancakes and syrup
C. Bagel with cream cheese
D. Cooked oatmeal and grapefruit half
Answer: D
Explanation:
(A) Eggs and hash browns do not provide much fiber and bulk, so they do not effectively
prevent constipation.
(B) Pancakes and syrup also have little fiber and bulk, so they do not effectively prevent
constipation.
(C) Bagel and cream cheese do not provide intestinal bulk.
(D) A combination of oatmeal and fresh fruit will provide fiber and intestinal bulk.
QUESTION 198
In cleansing the perineal area around the site of catheter insertion, the nurse would:
A. Wipe the catheter toward the urinary meatus
B. Wipe the catheter away from the urinary meatus

C. Apply a small amount of talcum powder after drying the perineal area
D. Gently insert the catheter another 12 inch after cleansing to prevent irritation from the
balloon
Answer: B
Explanation:
(A) Wiping toward the urinary meatus would transport microorganisms from the external
tubing to the urethra, thereby increasing the risk of bladder infection.
(B) Wiping away from the urinary meatus would remove microorganisms from the point of
insertion of the catheter, thereby decreasing the risk of bladder infection.
(C) Talcum powder should not be applied following catheter care, because powders
contribute to moisture retention and infection likelihood.
(D) The catheter should never be inserted further into the urethra, because this would serve no
useful purpose and would increase the risk of infection.
QUESTION 199
A 7-year-old child is brought to the ER at midnight by his mother after symptoms appeared
abruptly. The nurse’s initial assessment reveals a temperature of 104.5°F (40.3°C), difficulty
swallowing, drooling, absence of a spontaneous cough, and agitation. These symptoms are
indicative of which one of the following?
A. Acute tracheitis
B. Acute spasmodic croup
C. Acute epiglottis
D. Acute laryngotracheobronchitis
Answer: C
Explanation:
(A) Clinical manifestations of acute tracheitis include a 2–3 day history of URI, croupy
cough, stridor, purulent secretions, high fever.
(B) Clinical manifestations of spasmodic croup include a history of URI, croupy cough,
stridor, dyspnea, low-grade fever, and a slow progression. The age group most affected is 3
months to 3 years.
(C) Three clinical observations have been found to be predictive of epiglottitis: the presence
of drooling, absence of spontaneous cough, and agitation. Epiglottitis has a rapid onset that is
accompanied by high fever and dysphagia.

(D) Clinical manifestations of acute laryngotracheobronchitis (LTB) include slow onset with
a history of URI, low-grade fever, stridor, brassy cough, and irritability.
QUESTION 200
A client tells the nurse that she has had a history of urinary tract infections. The nurse would
do further health teaching if she verbalizes she will:
A. Drink at least 8 oz of cranberry juice daily
B. Maintain a fluid intake of at least 2000 mL daily
C. Wash her hands before and after voiding
D. Limit her fluid intake after 6 PM so that there is not a great deal of urine in her bladder
while she sleeps
Answer: D
Explanation:
(A) Cranberry juice helps to maintain urine acidity, thereby retarding bacterial growth.
(B) A generous fluid intake will help to irrigate the bladder and to prevent bacterial growth
within the bladder.
(C) Hand washing is an effective means of preventing pathogen transmission.
(D) Restricting fluid intake would contribute to urinary stasis, which in turn would contribute
to bacterial growth.

Document Details

Related Documents

person
Harper Mitchell View profile
Close

Send listing report

highlight_off

You already reported this listing

The report is private and won't be shared with the owner

rotate_right
Close
rotate_right
Close

Send Message

image
Close

My favorites

image
Close

Application Form

image
Notifications visibility rotate_right Clear all Close close
image
image
arrow_left
arrow_right